Articles and Readings Exercise Feedback – Early … and Readings Exercise Feedback – Early...

76
Articles and Readings Exercise Feedback – Early Childhood Education Module 1: Educational Administration as an Emerging Profession Article 1 Title: The Changing Shape of Leadership Questions: 1. How has instructional leadership changed? 2. What are the essential tasks of leadership identified in this article? 3. What is your own definition of authentic, instructional leadership? Question Responses: Responses should contain information from the following sections or boldfaced sections of text. 1. Participants should explain that The roles of principals, superintendents, and other education leaders have expanded during the past decade to include a larger focus on teaching and learning, professional development, data-driven decision making, and accountability (Institute for Educational Leadership, 2000). With leadership for student learning as the priority, instructional leadership might simply be described as “anything that leaders do to improve teaching and learning in their schools and districts.” 2. The following tasks that superintendents and principals engage in should be listed: Lead Learning Today's principals and superintendents are learning leaders: They participate in regular, collaborative, professional learning experiences to improve teaching and learning. Focus on Teaching and Learning Instructional leaders maintain and model a focus on improving teaching and learning by helping teachers improve their instructional practices and by making student achievement the highest priority. Develop Leadership Capacity Today's education leaders devote significant time to developing instructional leadership capacity in others in their schools or districts. Create Conditions for Professional Learning The new generation of instructional leaders actively create conditions that encourage professional learning communities.

Transcript of Articles and Readings Exercise Feedback – Early … and Readings Exercise Feedback – Early...

Articles and Readings Exercise Feedback – Early Childhood Education Module 1: Educational Administration as an Emerging Profession Article 1 Title: The Changing Shape of Leadership Questions:

1. How has instructional leadership changed? 2. What are the essential tasks of leadership identified in this article? 3. What is your own definition of authentic, instructional leadership?

Question Responses: Responses should contain information from the following sections or boldfaced sections of text. 1. Participants should explain that

The roles of principals, superintendents, and other education leaders have expanded during the past decade to include a larger focus on teaching and learning, professional development, data-driven decision making, and accountability (Institute for Educational Leadership, 2000). With leadership for student learning as the priority, instructional leadership might simply be described as “anything that leaders do to improve teaching and learning in their schools and districts.”

2. The following tasks that superintendents and principals engage in should be listed:

Lead Learning Today's principals and superintendents are learning leaders: They participate in regular, collaborative, professional learning experiences to improve teaching and learning.

Focus on Teaching and Learning Instructional leaders maintain and model a focus on improving teaching and learning by helping teachers improve their instructional practices and by making student achievement the highest priority.

Develop Leadership Capacity Today's education leaders devote significant time to developing instructional leadership capacity in others in their schools or districts.

Create Conditions for Professional Learning The new generation of instructional leaders actively create conditions that encourage professional learning communities.

Use Data to Inform Decisions In the current context of performance standards and accountability, instructional leaders know they must develop the skills to collect and use data from a variety of sources to inform school improvement decisions.

Use Resources Creatively Instructional leaders make creative use of all resources—people, time, and money—to support school improvement.

3. Answers will vary, but should include improving teaching and learning. Article 2 Title: The Power of Gentleness Questions:

1. After reading this article, do kindness and compassion seem compatible with school administration? Explain your response.

2. The author claims that a rigid leader fosters conflict. Can too much consistency lead to uncompassionate leadership? Can too many allowances lead to inconsistent leadership? Reflect on these differing positions and share your thoughts.

3. After reading the “Myths of Kindness & Compassion,” what myth have you found to be the most untrue? Relate your experience.

Question Responses: Responses should contain information from the following sections or boldfaced sections of text. 1.- 3. Responses will vary. Article 3 Title: Leadership the Sparks Learning Questions:

1. What are the major findings of the McREL study? 2. Reflect upon the most effective school leader you have known, which of the 21

key areas of leadership responsibility did that leader exhibit? 3. There are two primary variables that determine whether leadership will have a

positive or negative impact on student achievement. What are they? 4. In reflecting upon the two variables, what practices can you employ as a leader to

ensure that you make a positive impact on student achievement? Question Responses: Responses should contain information from the following sections or boldfaced sections of text. 1. Responses should contain the following:

• Leadership matters. McREL found a significant, positive correlation between effective school leadership and student achievement.

• We can empirically define effective leadership. McREL identified 21 key areas of leadership responsibility that are significantly correlated with student achievement.

• Effective leaders not only know what to do, but how, when, and why to do it. McREL researchers concluded that effective leaders understand which school changes are most likely to improve student achievement, what these changes imply for both staff and community, and how to tailor their leadership practices accordingly.

2. Participants’ responses will vary but should include several of the 21 areas listed. 3. Responses should state that

The first is the focus of change—whether or not leaders properly identify the correct focus for school and classroom improvement efforts, those that are most likely to have a positive impact on student achievement in their schools. The second variable is whether or not leaders properly understand the magnitude or order of change that they are leading and adjust their leadership practices accordingly.

4. Responses will vary but should relate directly to the focus of change and order of change in the previous response.

Module 2: Leadership Article 1 Title: Of Hubs, Bridges, and Networks Questions:

1. According to the author, why has the hierarchical model of leadership failed in affecting change?

2. What is your understanding of how communication operates within the network framework?

3. What “Practical Strategies for Using Networks to Bring Change,” as listed in this article, would be most effective for you to use in your school situation? Identify at least two.

Question Responses: Responses should contain information from the following sections or boldfaced sections of text. 1. Participants should summarize their response related to:

The failure of hierarchical models is understandable when we consider how we form human belief systems. Most leadership initiatives assume that, as rational beings, people will listen to the evidence, learn the new procedures, and follow directions. Leaders find it maddening when, even after they present the evidence and clearly teach new procedures, staff members don't implement the changes.

But changes in behavior do not follow the creation of a personal belief system; they precede it. Behavior does not stem from a rational consideration of evidence, but from an emotional attachment to a trusted colleague. Most workers do not feel this kind of attachment to higher-ups. In a 2004 survey, the Gallup Organization found that 55 percent of adult workers did not feel connected to the organizations that employed them—and 16 percent felt actively disengaged from their organizations (B. Sanford, personal communication, Aug. 11, 2005). Kouzes and Posner (2003) found high levels of distrust of leadership in organizations of all kinds. Leaders can certainly endeavor to become more credible, open, and trustworthy. But employee disengagement seems to be more an inherent characteristic of organizations than a reflection of poor leadership.

Gallup found that the majority of employees take their cues from a trusted colleague rather than from the boss, the employee manual, or a silver-tongued trainer (Rath & Clifton, 2004). In other words, even the best leaders cannot transform islands of excellence into systemic change by relying on better bosses, clearer employee manuals, or more eloquent trainers. The delusions of strategic plans and management charts notwithstanding, organizations function not as hierarchies, but as networks.

2. Participants should include and explain the following boldface words in their response to demonstrate understanding:

Rather than assuming that leadership can distribute a message in a linear manner down through the hierarchy of an organization, the network framework suggests that change spreads throughout the system on a distinctly nonlinear communication path of nodes, hubs, and superhubs. A node is any single point of contact in a network; within an organization of people, a node is one individual. A hub is a node with multiple connections to other nodes. Within a school, a hub could be a grade-level leader, a department chair, or a principal. A superhub is that rare node in a network to which an exceptionally large number of other nodes and hubs are connected. In a school, the superhubs do not necessarily hold administrative titles, but they are those to whom colleagues come for advice. The superhub gets 20 times more e-mail than colleagues do. She or he receives regular classroom visits and informal observations from friends and colleagues.

3. Participants should relate at least two or more of these strategies to their present school situation and relate its use to bringing change.

Practical Strategies for Using Networks to Bring Change

• Select one struggling student and one gifted student to observe intently. Keep weekly journals on instructional practice. What worked? What didn't? What's next? Read from these journals at faculty meetings.

• Create an annual “Best Practices Book” every year, with each teacher contributing a single page. Give all teachers a copy of the booklet at the end of each year, and give it to new teachers as a welcome gift.

• Survey students about which teachers they would seek out for help. Ask students to identify specific teaching practices that have aided their learning.

• Some teaching cultures regard openly sharing success stories as self-promotion. Allow anonymous sharing: Before each faculty meeting or professional development session, give each teacher a form on which to anonymously record challenges and success stories. Compile the best success stories into a PowerPoint presentation.

• Focus meetings on concrete discussion of practices or questions. Make it clear that negative comments (“That will never work with my kids”) are out of bounds.

• Arrange seats in a circle or horseshoe at meetings, so there is no back row from which “Jack the Jerk” can safely lob toxic bombs.

• Make staff development voluntary—a radical step, but worth considering. Negative people actively interfere with the learning opportunities of others and do not apply the learning themselves.

• Create a rubric that clearly defines “jerk” behavior. Some toxic hubs think their acerbic comments and put-downs are funny. But professionals should not tolerate bullying within a faculty meeting any more than they would on the playground.

Article 2 Title: Improving Relationships Within the Schoolhouse Questions:

1. Do you agree or disagree with the author’s claim that, “the relationships among the educators in a school define all relationships within that school's culture.” Back up your assessment with information from the article and your own experience.

2. Four forms of relationships are described among adults within the schoolhouse: parallel play, adversarial relationships, congenial relationships, and collegial relationships. Do these exist in reality? Can you think of at least one or two general examples that you have experienced or seen and account for why these relationships exist as they do?

3. Explain what must occur to establish a culture of collegiality focusing on the school leader’s role in its achievement.

Question Responses: Responses should contain information from the following sections or boldfaced sections of text. 1. Participants’ responses should reflect the author’s position as stated below and convey agreement or disagreement with this position.

The nature of relationships among the adults within a school has a greater influence on the character and quality of that school and on student accomplishment than anything else. If the relationships between administrators and teachers are trusting, generous, helpful, and cooperative, then the relationships between teachers and students, between students and students, and between teachers and parents are likely to be trusting, generous, helpful, and cooperative. If, on the other hand, relationships between administrators and teachers are fearful, competitive, suspicious, and corrosive, then these qualities will disseminate throughout the school community.

2. Responses can relate examples to parallel play (working in isolation), adversarial relationships, congenial relationships (personal/friendly), or collegial relationships (professional learning community). 3. Responses may include any of the following: talking about practice, sharing craft knowledge, observing one another, and rooting for one another to what school leaders can do.

Researcher Judith Warren Little found that school leaders foster collegiality when they

• State expectations explicitly. For instance, “I expect all of us to work together this year, share our craft knowledge, and help one another in whatever ways we can.”

• Model collegiality. For instance, visibly join in cheering on others or have another principal observe a faculty meeting.

• Reward those who behave as colleagues. For instance, grant release time, recognition, space, materials, and funds to those who collaborate.

• Protect those who engage in these collegial behaviors. A principal should not say, for instance, “Janet has a great idea that she wants to share with us today.” This sets Janet up for a possible harsh response. Rather, the principal might say, “I observed something in Janet's classroom last week that blew my socks off, and I've asked her to share it with us.” In this way, leaders can run interference for other educators.

Article 3 Title: Starting with the Soul Questions:

1. What is the basis for the authors’ position on effective professional development and Maslow’s pyramid?

2. Relate one or more of the “Models of Successful Growth” cited by the authors to your own beliefs about professional development.

3. The authors believe that programs, such as Courage to Teach, can “. . . renew and deepen an educator's sense of purpose . . . to explore what is important about his or her work.” Based on your experience with professional development, do you see a need for this in professional development? Why or Why not?

Question Responses: Responses should contain information from the following sections or boldfaced sections of text. 1. Participants should explain:

We believe that effective professional development and the powerful teaching it can cultivate require an inversion of Maslow's pyramid. We need to begin with the soul of the enterprise, the passion and purpose that animate teachers' ongoing commitment to students and learning. Too many of our conversations about effective teaching focus on content, teaching methods, and learning outcomes. We neglect teachers themselves, the individuals who step in front of students each day committed to engaging them in the unpredictable and challenging process of learning.

2. Participants should articulate a personal philosophy related to any of the following

Peter Senge's influential book Schools That Learn (2000) …schools must become organizations staffed by individuals who know how to learn and grow. Senge lays out the five disciplines of an organization that learns: personal mastery, shared vision, mental models, team learning, and systems thinking.

Two organizational researchers from Stanford (Collins & Porras, 1994) who examined what distinguishes an organization that is merely “good” from one that has achieved enduring success discovered that collective passion and mission were at the core of the latter kind of organization.

Andy Hargreaves and Michael Fullan (1998) speak more specifically about reforming the profession of teaching. They argue that education policymakers and administrators typically overlook the role that a teacher's sense of purpose plays in education change. . . . Hargreaves and Fullan advocate going deeper, leading teachers into

hard thinking and soul searching about the fundamental value and purpose of what we do as educators.... Going deeper, in other words, involves purpose, passion and hope. (p. 29)

3. Responses will vary.

Module 3: Decision Processing and Decision Making Article 1 Title: Creating a Community of Difference Questions:

1. Explain the author’s definition of communities of difference. 2. The author states, “To create communities of difference grounded in social justice

and academic excellence, education leaders and community members together must develop shared understandings around certain criteria to guide both short-term and longer-term decision making—criteria such as justice, caring, democracy, and optimism.” Explain how these principles are infused into the decision making process of a school.

3. After reading this article, formulate your response to the question posed initially by the author, “How should a leader lead in tough times?”

Question Responses: Responses should contain information from the following sections or boldfaced sections of text. 1. Participants should explain that

Although we generally think of community in terms of what binds participants together—shared norms, beliefs, and values—communities of difference are based not on homogeneity but on respect for difference and on the absolute regard for the intrinsic worth of every individual. Members of such communities do not begin with a dominant set of established norms but develop these norms together, with openness and respect, as they share their diverse perspectives.

2. Participants should relate the criteria given to the decision making process to arrive at the main premise that…

An education leader who wants to create a community of difference will use that power deliberately and morally to promote meaningful relationships and deep understanding. A primary way to accomplish these goals is through dialogue.

Other points can include the following:

The deliberate and moral creation of a community of difference requires building relationships with each member of that community. Education that is just and caring attends to both the contexts and the outcomes of the learning experience. . . . Just and caring education addresses the needs, abilities, and interests of all students by offering a range of programs, including advanced classes and college prep and vocational and technology programs. Moreover, access is free and open to all students. Education leaders must help their staff explore how to develop such environments and understand the implications of working with parents and community

members. We cannot simply teach facts; we must encourage students to bring their own experiences to the sense-making conversation.

Education that is democratic and optimistic opens doors of opportunity to all students. A democratic education includes safeguards that protect minority rights. Hence, an education leader does not simply issue an invitation to participate but makes certain that everyone knows what this participation entails. A community of difference is inherently optimistic. It includes all students and their families regardless of their backgrounds. It legitimizes the experiences of those whose lives are different from those assumed by dominant norms and cultural values. . . . A democratic education is optimistic because it permits the full participation of all students and prepares them equally for choices and challenges beyond school.

3. Answers will vary. Article 2 Title: Leadership for Lasting Reform Questions:

1. Explain what is meant by high leadership capacity. 2. Identify the principal’s characteristics that you determine to be critical in

developing leadership capacity in others. 3. The author identified three major phases of development--instructive, transitional,

and high capacity—leading to teachers taking on leadership roles. After reading each phase, where would you place your school and why?

Question Responses: Responses should contain information from the following sections or boldfaced sections of text. 1. High capacity leadership should be defined as “skillful participation in the work of leadership.”

2. Participants can identify any of the following:

The principals shared certain characteristics that contributed to their schools' evolving culture of leadership, including

• Understanding of self and clarity of values; • A strong belief in equity and the democratic process; • Strategic thought about the evolution of school improvement; • A vulnerable persona; • Knowledge of the work of teaching and learning; and • The ability to develop capacity in colleagues and in the organization.

3. Responses will vary.

Article 3 Title: The Most Important Data Questions:

1. Explain how the walk-through was conducted and evolved to generate valuable data in this school district.

2. Summarize the process that occurs in the fall to determine the focus area for the year.

3. What are several of the results and benefits of this process?

Question Responses: Responses should contain information from the following sections or boldfaced sections of text. 1. Respondents should explain that

A walk-through is an organized school or classroom tour focused on teaching and learning. The walk-through process developed by Resnick focuses on looking at student work and talking with students to determine what they can express about their learning. We realized that this strategy could hold the key to school improvement.

2. Participants should include the key points as highlighted:

In the fall, each school conducts the first walk-through of the year. Administrators, teachers, and invited community members walk through the school, interviewing every student about his or her learning. The process varies from school to school. In most cases, the school sets up desks in the hallway and pulls students out of their classrooms individually to be interviewed. A typical interview lasts about five minutes. All students can be interviewed within a few hours if the school can sign up and train enough interviewers.

The teacher committee collects the data sheets and meets to analyze student responses and to target priority areas for improvement. Usually the committee meets the same day and tallies the data in an hour or two, either manually or by computer. At the next faculty meeting, the committee provides the teaching staff with the results and its recommendations. The school now has a focus for the year, and teachers begin in earnest to work on the committee's recommendations. The following spring, each school conducts a second walk-through to determine how much growth the school has made in the focus area.

3. Respondents may include the following:

Walk-throughs have, first and foremost, transformed our district's schools by bringing into sharper focus not only what teachers are teaching but also what students are learning. This, in effect, is a shift toward a more results-oriented school district.

Second, teachers and administrators have gained a new insight: It doesn't matter how well a lesson plan is written or how well the lesson is taught if students cannot clearly articulate the lesson's content and purpose.

Third, walk-throughs provide the most important data of all—the data closest to the students. The process produces authentic, fresh, clear information that gives teachers immediate guidance about where they must direct their efforts. . . . Yet it empowers students, teachers, and administrators, and as a result, the entire school. Schools use walk-through data to direct their plans for improving instruction. Professional staff development for the year is integrally tied to the walk-through process.

Module 4: School Improvement Through Systematic Planning Article 1 Title: Having It All Questions:

1. Compare your knowledge of the terms “teacher-centered” and “student-centered” and contrast this with the enactment of teacher-centered and student-centered practices at Hersey & Northtown.

2. How do these schools produce success with students who enter with serious skills deficiencies without these same students experiencing the stigma often associated with remediation and ability grouping?

3. According to this article, the educators at Hersey and Northtown have found a way to blend their “pedagogical and ideological differences into a coherent curricular structure. The approach is carefully planned and calibrated . . .” Briefly explain how this was done and its results.

Question Responses: Responses should contain information from the following sections or boldfaced sections of text. 1. Participants should compare their prior knowledge of the definitions of teacher- and student-centered practices in contrast with the new interpretation of teacher- and student-centered practices at Hersey & Northtown.

It may seem paradoxical at first to use the term “student-centered” to describe a model that focuses on building students' skills in alignment with standardized assessments. In conventional professional usage, the term usually refers to curricular practices that start with individual student interests and aim to cultivate diverse individual talents. In contrast, schools in which teachers determine the content and pacing of the curriculum tend to be derogated as “teacher-centered.”

But at Hersey and Northtown, these terms have a different resonance. There, “teacher-centered” refers to school policies that permit individual teachers to teach idiosyncratically without a collective plan for ensuring that all students succeed according to measurable criteria. “Student-centered” means that teachers coordinate and align their efforts to ensure that students master essential skills and knowledge.

2. Participants should convey that: The way Hersey and Northtown target remediation is the model's most controversial feature: They group students by skill level. This enables teachers to address students' needs with greater concentration and precision than they could in heterogeneously grouped classrooms. However, all students study the same content and participate collectively four times a year in the cohortwide integrated units. Requiring all students to follow a common core curriculum mitigates the stigma associated with remediation and ensures that all students grapple with the same content and issues.

3. Participants should convey a comprehensive understanding of the planning and curricular structure employed within the model presented, to include the following:

Teachers must accept certain ground rules—beginning with an agreement to embrace standardized testing. Its success depends on teachers' willingness and ability to do the thankless, mundane work of skills diagnosis and instruction alongside more intrinsically rewarding tasks, such as crafting debates over genetic research.

They start with the College Readiness Standards in mapping what they need to teach students. They use both ACT-produced and teacher-created diagnostic assessments to place students, tailor instruction, and target remediation. Students also have a strong participatory role to play . . . while students have no formal role in shaping the basic structure of the curriculum, and as a result of the consolidation of course offerings, they have fewer curricular choices. But the interdisciplinary projects that cap each integrated unit provide one opportunity for students to take ownership of the content and to practice self-directed learning. The public forums provide another. These are organized like town halls, and adults and students alike prepare for and participate in them, forming a community of learners pursuing focused inquiry. Here, students get the opportunity to practice the crucial citizenship skills of reasoned public argumentation and shared deliberation.

The schools break the molds of both traditional and innovative theories of schooling and reintegrate these theories to powerful effect. Teaching and learning in these schools are standards-based and student-centered, broad and deep, subject-focused and integrative, canonical and critical, academic and civic in orientation. Both schools enhance teacher and student freedoms by judiciously constraining them and cultivate students' critical reasoning by providing them with consequential material to reason about. And students have responded, through both spirited engagement in their learning and high performance on standardized achievement measures.

Article 2 Title: Adding Value to Accountability Questions:

1. Explain the assumptions that value-added analysis makes in seeking to determine, “How much value has a school added to a student's learning?”

2. What is some of the information that value-added analysis supplies, and what are the benefits of this information?

3. After reading this article and the opposing views presented, what is your position on the use of value-added analysis?

Question Responses: Responses should contain information from the following sections or boldfaced sections of text. 1. Responses should contain explanations of the two assumptions:

Value-added analysis makes two important assumptions. First, it assumes that one can measure an individual's growth in learning from one measured occasion to the next. Therefore, it tracks the progress of individual students over time. These longitudinal measures differ from conventional measures because they promote growth for all students rather than focusing on status.

Value-added analysis also assumes that schools have only partly contributed to changes in test scores. Learning is the sum of many factors, including both school and nonschool elements. School evaluations need statistical methods to separate the impact of the school from the impact of nonschool characteristics, such as students' socioeconomic background. For this reason, value-added analysis relies on advanced statistical methods to estimate the effects of the school.

2. Participants should include that: Value-added analysis statistically estimates the contribution of the school as separate from the non-school-related variables, such as economic status, that contaminate traditional analyses. Benefit: Value-added analysis, combined with other valid indicators, can more reliably assess school quality without punishing or rewarding schools for preexisting differences related to student background characteristics or other non-school-related factors.

Value-added analysis can assess both the percentage of students at the proficient level and the progress schools are making toward the proficiency standard (Thum, in press).

Benefit: Only when educators examine longitudinal trends of individual students, relating them to reference groups and outcome targets, can they tailor instructional strategies to meet the needs of individual students.

3. Responses will vary.

Article 3 Title: A Blueprint for Increasing Student Achievement Questions:

1. What are the three strategies employed by the authors to systematically address student achievement?

2. Explain each of the three strategies and their benefits. 3. How is the principal’s role as instructional leader critical to this process? 4. The authors state, “Our mission is student success.” How does your school

address this mission and what strategies discussed in this article are used or could be employed to facilitate the achievement of this mission in your school?

Question Responses: Responses should contain information from the following sections or boldfaced sections of text. 1. Responses should contain the following three strategies:

Teachers design pacing guides as frameworks for instructional planning (strategy 1) and formative assessments that reveal what students have learned (strategy 2). Then, at regularly scheduled times during the school year, teachers come together in staffing meetings (strategy 3) to discuss their students' progress, analyze formative assessment data, generate ideas to assist struggling students, and gain support from one another and from administrators.

2. Each of the strategies should be defined (see # 1 above) and its benefit listed as follows:

A pacing guide is a standardized format for long-range planning that groups learning objectives into units, allocates time to each unit, and sequences units on a calendar. A pacing guide provides a plan for teaching the relevant curriculum before administering the state-mandated assessments.

In designing formative assessment tools, teachers should pay close attention to alignment. Does the assessment match the learning goals on the pacing guide and the standards being assessed? If so, then the assessment should have some predictive quality, at least enough to reveal which students need additional instruction and in which areas. Teachers make plans for addressing these student needs in staffing meetings.

The power of formative assessment increases when teachers of the same content area design and administer common assessments. As teams of teachers analyze the results of common assessments, the progress of students becomes a schoolwide concern—the process becomes more systematic and less variable.

The creation of pacing guides and the collection of formative assessment data become nothing more than busywork if a formalized process for periodically reviewing students' progress and updating the guides is not in place. We recommend holding staffing meetings at least three times a year to monitor progress on instruction as described in the pacing guide. In these meetings, teachers review formative assessment data; revise pacing guides as needed; identify problem learning areas for the group as a whole, for small groups, or for individual students; and plan appropriate interventions and differentiation of curriculum. Pacing guides are living documents, and staffing meetings provide opportunities to continually revise and refine them.

3. Respondents should explain that the principal. . .

sets the tone for the task by emphasizing its importance and placing the work within the context of the overall school improvement process. Principals move the work ahead by providing both positive pressure and meaningful support. Positive pressure comes in the form of deadlines, feedback, and an expectation of accountability. Meaningful support includes providing technical assistance, clerical help, time, and other resources.

Principals can exhibit instructional leadership by introducing the process, participating in and monitoring the writing of pacing guides, coordinating a production timetable, hiring technical support persons such as district-level

subject-area specialists, and allocating clerical support so that teachers' time is not spent typing documents. The principal may delegate certain leadership duties to grade-level leaders (elementary level) and department chairs (secondary level), but he or she must be the prime mover of this project. Guiding this process is instructional leadership at its best.

4. Responses will vary.

Module 5: The School Curriculum Article 1 Title: Creating a Differentiated Mathematics Classroom Questions:

1. Explain the four mathematical learning styles and when and how they emerge. 2. After reviewing the learning styles and the specific instructional approaches, how

can a teacher incorporate this information into the teaching of mathematics? 3. How can a teacher then differentiate based on Tomlinson’s recommendations?

Question Responses: Responses should contain information from the following sections or boldfaced sections of text. 1. Participants’ responses should contain the following:

Differences in students' mathematical learning styles emerge quite early in their development. . . . Students can acquire and make use of more than one style. Although people can work in all four styles, we generally tend to develop preferences or strengths in one or two of the styles and develop weaknesses in the ones that remain.

• The Mastery style: People in this category tend to work step-by-step.

• The Understanding style: People in this category tend to search for patterns, categories, and reasons.

• The Interpersonal style: People in this category tend to learn through conversation and personal relationship and association.

• The Self-Expressive style: People in this category tend to visualize and create images and pursue multiple strategies.

2, Responses should contain information related to rotating strategies, using flexible grouping, personalizing learning, and developing fair and thoughtful testing practices. 3. Respondents should convey the following recommendations:

• Include all four dimensions of mathematical learning—computation, explanation, application, and problem solving—in every unit we teach;

• Help students recognize their own mathematical learning styles—Mastery, Understanding, Interpersonal, or Self-Expressive—along with their strengths, their weaknesses, and where they need to grow;

• Use a variety of teaching strategies to explore mathematical topics; and

• Create or revise our assessments to reflect all four dimensions of mathematical learning and all four learning styles that students use to approach those dimensions.

Article 2 Title: Becoming an Engaged Reader Questions:

1. The authors state, “We want to make two promises to every child: We will teach you to read, and we will help you become a reader—a literate person who experiences the power and joy of comprehending.” How are learning to decode and learning to comprehend inseparable in creating a reader?

2. How does an interactive read-aloud create a community of readers? 3. What is guided reading and how is it structured? 4. How does emotion influence reading behaviors and reading development?

Question Responses: Responses should contain information from the following sections or boldfaced sections of text. 1. Participants should explain that

Learning to comprehend is an ongoing process, a thinking process that expands across time as the individual encounters different texts, in different ways, for different purposes. Students do not first learn to decode and then become readers; they must be engaged in reading, thinking about, and discussing interesting texts from the beginning. Some important insights about readers, teachers, texts, and emotions can help foster this kind of learning.

2. Responses should state that interactive read-alouds,

Usually taking place in whole-group settings, interactive read-aloud is an efficient way for students to expand their vocabulary and concepts and share and understand texts. Through interactive read-aloud, teachers . . .

• Teach students how to talk with one another about texts.

• Provide meaningful, enjoyable group experiences.

• Give students opportunities to process language and think about texts that are too difficult for most of them to read independently.

• Engage readers in thoughtful discussion.

• Model and provide group support for fluent, phrased reading.

3. Participants’ responses should contain the following information: Guided reading is small-group instruction for students who exhibit similar reading behaviors and who read at similar levels. The teacher selects a text and introduces it; then each student reads the text either softly or silently. The teacher observes, notes students' reading behaviors, and sometimes interacts briefly with individuals. After the reading, students discuss the meaning of the story, and the teacher helps students practice processing strategies and engages the students in phonics/word study work. Guided reading may also include extending the text through writing, drawing, discussion, drama, or another kind of analysis (Fountas & Pinnell, 1996).

4. Responses should contain any of the following information relating emotion to reading behaviors and reading development:

Neuroscientists have proven that reading comprehension is a complex and individual constructive process. They have also found that beginning readers create networks in their brains to link what they see on the page to the language they speak. Emotions organize the neuronal networks required to think, retrieve previously learned information, understand, and remember (Ratey, 2001).

No matter how a child reacts to the learning situation, however, adults' responses affect the child's emotional, social, and cognitive development (Lyons, 2003a). . . .If children repeatedly sense disapproval, they are likely to remember negative experiences and avoid putting themselves in those circumstances again. . . . Children who are experiencing difficulty learning to read commonly become frustrated. Continued emotional distress can create deficits in a child's intellectual abilities, crippling his or her capacity to learn (Levine, 2002). That is why we need to look for and support children's approximations or partially correct responses.

Carol, David's Reading Recovery teacher, noted that their relationship had contributed to his learning. Carol demonstrated a genuine interest in David and his learning, which helped change his attitude and increase his interest in reading. Carol also purposefully selected books and writing topics that built on David's strengths and interests, which further motivated him to engage as a reader and writer.

The brain's organization reflects its experience. If the child's experience is characterized by fear, anxiety, stress, and helplessness, then the chemical responses to these emotions become the most powerful architects of the brain (Damasio, 2003). Fortunately, emotionally positive learning experiences can change children's attitudes and provide motivation to learn.

Article 3 Title: You Can Teach for Meaning Questions:

1. What key principles are embodied in teaching for meaning and understanding? 2. In the two examples given, identify these key principles. 3. Cite one research-based example provided that refutes “Misconception 1: We

Have to Teach to the Test.” 4. Explain how “uncoverage”—focusing on fewer topics and core understandings—

is more likely to increase student achievement. Question Responses: Responses should contain information from the following sections or boldfaced sections of text. 1. Respondents should list:

• Understanding big ideas in content is central to the work of students.

• Students can only find and make meaning when they are asked to inquire, think at high levels, and solve problems.

• Students should be expected to apply knowledge and skills in meaningful tasks within authentic contexts.

• Teachers should regularly use thought-provoking, engaging, and interactive instructional strategies.

• Students need opportunities to revise their assignments using clear examples of successful work, known criteria, and timely feedback.

2. Participants should select specific information from the two examples that address each of the key principles. Responses may vary. 3. Responses will vary based on the example selected. 4. Participants should explain that uncovering the curriculum, while offering fewer topics, enables a more coherent and focused content.

This concentrated focus enabled teachers and students to gradually build more complex understandings in mathematics, to delve deeply into subject matter, and to attain higher levels of achievement (Schmidt, 2004; Schmidt, Houang, & Cogan, 2002). Using this “uncoverage” approach—focusing on fewer topics and core understandings—teachers can avoid the twin problems of aimless coverage and activity-oriented instruction.

Module 6: Promoting Student Achievement Through Positive Learning Communities

Article 1 Title: Creating a School Community Questions:

1. What basic psychological needs are met in a school with a strong sense of community?

2. What are some of the lasting benefits for students in a school with a strong sense of community?

3. In looking at the four approaches that are needed to build community, why do you think so few schools have this strong sense of community?

4. What activities that are given in the examples from The Child Development Project would be reasonable first steps in establishing or strengthening a sense of community in your school?

Question Responses: Responses should contain information from the following sections or boldfaced sections of text. 1. Participants should recount the specific basic psychological needs that are met in a school with a strong sense of community, such as:

These questions reveal our basic psychological needs—for emotional and physical safety; for close, supportive relationships—a sense of “connectedness” (Resnick et al., 1997) or “belongingness” (Baumeister & Leary, 1995); for autonomy, or a say in what happens to us; and for a sense of competence—a belief that we are capable people and able to learn. These fundamental needs shape human motivation and have major implications for learning and development. We are willing to work very hard to preserve our sense of safety, belonging, autonomy, and competence (Deci & Ryan, l985).

We also bond with the people and institutions that help us satisfy our needs (Watson, Battistich, & Solomon, 1997), which makes the creation of caring, inclusive, participatory communities for our students especially important. When a school meets students' basic psychological needs, students become increasingly committed to the school's norms, values, and goals. And by enlisting students in maintaining that sense of community, the school provides opportunities for students to learn skills and develop habits that will benefit them throughout their lives.

2. Participants should include several of the following:

Students in schools with a strong sense of community are more likely to be academically motivated (Solomon, Battistich, Watson, Schaps, & Lewis, 2000); to act ethically and altruistically (Schaps, Battistich, & Solomon, 1997); to develop social and emotional competencies (Solomon et al., 2000);

and to avoid a number of problem behaviors, including drug use and violence (Resnick et al., 1997).

3. Participants should base their evaluation on one or more of the following:

Actively cultivate respectful, supportive relationships among students, teachers, and parents. Supportive relationships are the heart of community. They enable students from diverse backgrounds to bring their personal thoughts, feelings, and experiences into the classroom. Supportive relationships help parents, especially those who would otherwise feel vulnerable or uncomfortable, take active roles in the school and in their children's education.

Emphasize common purposes and ideals. Along with academic achievement, schools with a strong sense of community stress the development of qualities essential to good character and citizenship, such as fairness, concern for others, and personal responsibility. Everyone shares an understanding of the school's values, which then shape daily interactions.

Provide regular opportunities for service and cooperation. Students learn the skills of collaboration, develop wider and richer relationships, and experience the many satisfactions of contributing to the welfare of others.

Provide developmentally appropriate opportunities for autonomy and influence. Having a say in establishing the agenda and climate for the classroom is intrinsically satisfying and helps prepare students for the complexities of citizenship in a democracy.

4. Participants should apply one or more of the following approaches to their own school situation:

• Class meetings are useful for setting goals and norms, planning activities, and identifying and solving problems. They are essential for building peer relationships and fostering shared goals in the classroom. Class meetings at the beginning of the year, for example, include a few “unity builders.” Students may bring a favorite toy or memento and discuss it with a partner, who then presents it to the entire class. The class later may collaboratively establish goals for the year (such as “To make our room a safe place for everyone”), shared values (such as “To treat one another with respect”), or shared norms (such as “To make decisions by consensus whenever we can”).

• A buddies program pairs whole classes of older and younger students for academic and recreational activities. Every older student gets a younger buddy for the year. They get acquainted by interviewing each other, charting ways in which they are alike and different, and sharing their classroom portfolios. During the year, they may read or play math games together, visit museums, work together for a cause, or create a joint journal of their activities. At year's end, they show their mutual appreciation by exchanging thank you notes or gifts that they have made. Buddies programs help create powerful cross-age relationships, teach important social skills, and create a caring ethos in the school.

• “Homeside” activities are short conversation activities for students and parents or other caregivers to do at home once or twice a month. These conversations,

mostly interviews conducted by students with their parents, link school learning with home experiences and perspectives. For 4th grade state history units, for example, students interview their parents about how their family or ancestors first came to their state. Whether family members have lived in the state for 100 years or 100 days, the story of how and why they came to settle there is part of state history and serves to personalize learning for students.

• Schoolwide community-building activities link students, parents, and teachers; help foster new school traditions; and promote helpfulness, inclusiveness, and responsibility. They can be as undemanding as Family Film Nights, invitations to the entire family to view a feature-length movie at school and perhaps discuss a question related to it within the family. Or the activities can be as challenging as creating a Family Heritage Museum, for which students and their caregivers prepare displays of information and artifacts that tell something about their family heritage. The school then features the displays for a week or two and organizes an evening event so that parents and children can view them together.

Article 2 Title: Welcome to the House System Questions:

1. Explain the social house system and describe how it functions. 2. What were the goals of implementing the house system in the Goleta Valley

Junior High? 3. Who was involved and how did this school implement this system? 4. Could a house system be applied at other levels—elementary, high school,

postsecondary? Support your evaluation with information from this article. Question Responses: Responses should contain information from the following sections or boldfaced sections of text. 1. Respondents should explain that …

social houses divide students into multiple social units rather than into separate academic entities. Each unit has its own identity and theme. At Goleta Valley, all students mix during the day in regular classrooms but divide into their four houses during social and academic competitions, community service activities, and schoolwide leadership meetings. The houses reflect the school's diversity, encompassing students of various races, ethnicities, ages, and academic abilities. Teachers and staff members are assigned to the houses to encourage stronger relationships between adults and students. Each house has approximately 230 students and 17 staff members. Once the houses are formed, they compete against one another for points and build a greater sense of community in the process.

2. Responses should convey the following goals:

Our primary goal was to use social houses as a means of building school community. We hoped that a smaller-feeling school would promote new friendships among students and help them develop citizenship skills, stronger

relationships with staff, and a greater sense of identity. A wide body of research suggested that improved school climate would also translate into better academic results in the classroom (Hoy & Hannum, 1997).

3. To explain how the system was implemented and who was involved, responses should contain the following information:

We actively solicited student involvement in designing the program. . . . A local artist created a logo for each house to place on posters, T-shirts, flags, and ID cards. Students selected the winning artwork.

Students, teachers, cafeteria employees, office staff, custodians, and even the security guard were assigned to one of the four houses. To help students and staff identify fellow members of their new social teams, we distributed T-shirts and campus IDs with house logos and displayed group pictures of the houses throughout the campus.

We communicated these goals [see # 2 above] to students at an opening assembly and described the program's point system. The house that earned the most points between September and June would earn a trophy called the Mariner Cup. Houses could accumulate points through acts of citizenship or achievement by individual students, participation in group competitions, and housewide contributions to the local community.

4. Responses will vary.

Article 3 Title: A View from the Classroom Questions:

1. Cite a specific example from each section—respecting students, communicating with students, and supporting students—and relate how these behaviors on the part of the principal can lead to a positive school climate.

2. How can a positive school climate have the potential to promote student achievement?

3. Write a statement attesting to the behaviors a school leader should practice to build a positive climate for student learning and achievement.

Question Responses: Responses should contain information from the following sections or boldfaced sections of text. 1. Participants can use any example given in these three sections and relate them to the creation of a positive school climate. 2. Responses will vary.

3. Responses will vary.

Module 7: Special Students and Special Services

Article 1 Title: A Public Agenda Survey/When It’s Your Own Child Questions:

1. Why can special education polarize schools and parents of special needs children? 2. What findings suggest that an information campaign may be one way of bridging

this polarization, but also lead to the premise that school leaders need to shift their perspective?

3. What are the five key findings that school leaders need to consider in dealing with special education policies and programs?

4. What are the implications of this study for school leaders, and could these findings guide your school’s special education practices?

Question Responses: Responses should contain information from the following sections or boldfaced sections of text. 1. The responses should restate the general findings of the survey, which are:

Our findings suggest that special education may be an issue that invites polarization. Parents of children with special needs want as much help as they can possibly get, but many school leaders believe that such programs take up too much time and money. Strong majorities of superintendents and principals claim that special education uses a disproportionate amount of money and resources (Johnson, Farkas, & Duffett, 2001).

2. Respondents should restate the findings regarding the lack of information parents possess and that school leaders need to look at special education from the parents’ point of view.

Public Agenda's research strongly suggests that parents of children with special needs are not well informed about the policy issues and federal mandates that challenge local educators. Only 29 percent of special education parents in our study understood that “much of what happens in special education takes place because of the federal government” (Johnson & Duffett, 2002, p. 27), and 63 percent readily admitted that they were not too familiar with IDEA. Although money is often the paramount worry for local school administrators, few parents in our study were aware of the costs of special education or of the disputes between the federal government and local districts over how much expense local schools should bear. . . .

A natural response might be to mount an information campaign to provide parents of special-needs students with details on the costs of special education and on the new IDEA legislation. Our study indicates that more information may be useful, but it also suggests that an arsenal of statistics is unlikely to change

the central point: Parents of special-needs students approach the issue from an entirely different angle than policymakers and administrators do. . . .

Most are attempting to be their child's advocate and protector. To make changes in special education that both ease the burden for educators and provide the services that families value, education leaders need to absorb the parents' point of view.

3. Participants should list the following: • From the point of view of most special-needs parents, special education is not

a failed government program. • Parents' biggest complaints about special education concern the difficulty of

getting information in the early stages—not the services that their children receive now.

• Most special education parents do not believe that schools and teachers push students into special education, although minority parents have more concerns.

• Parents complain that schools should expect more academically from special-needs students, but most also want accommodations in testing.

• There is a group of extremely frustrated and dissatisfied parents who fear that schools are failing their children.

4. Participants should focus on the following implications and relate them to their schools’ special education practices.

One clear implication is that leaders are dealing with two different special education constituencies. Parents who believe that they and their children have been well served by special education may be quite willing to discuss how special education works and to consider whether schools could provide good services at more affordable costs. On the other hand, parents whose experiences with special education have produced mainly frustration and anger may see such overtures as one more attempt to keep them from getting the help that they believe their children need.

Another policy implication that emerges from the data is the extent to which the concerns of both critics and parents center on identification and diagnosis. There is a widespread sense among special education parents that not every diagnosis is valid and that special education is not the solution to every problem. School leaders should ask themselves whether they need to strengthen the process of identifying which children need special education services, and, further, whether parents whose children may have behavior problems rather than genuine learning disabilities have any other alternatives for getting help. How we determine which students need special education services—and which kind of services will help them—is an area that bears a second look.

Article 2 Title: Connecting With Latino Learners Questions:

1. Define what groups the term Latino refers to and what is likely to influence Latinos’ acquisition of English if they are English Language Learners (ELL).

2. What does it mean when educators build on Latinos’ existing funds of knowledge. 3. Describe at two effective strategies that can help scaffold content-area instruction

for ELL students in the process of developing academic English.

Question Responses: Responses should contain information from the following sections or boldfaced sections of text. 1. Participants should define Latino as including

Mexicans, Central Americans, Puerto Ricans, Dominicans, and South Americans. This is not a homogeneous group. In fact, Gutierrez and Rogoff (2003) warn educators against pigeonholing individuals.

Despite what many teachers may believe, an English language learner's IQ, country of origin, ethnicity, or culture does not determine the rate at which he or she develops academic English. Although socioeconomic status is likely to influence students' opportunity structures—the neighborhoods in which they live, the schools they attend, and their interactions and participations in particular activities—Gutierrez and Rogoff (2003) remind educators that students' circumstances should not be viewed as “traits.”

2. Responses should explain that . . .

teachers have incorporated into their curriculums Latino families' funds of knowledge—the lived experiences and informal bodies of information that exist in homes, communities, and the families' social networks. These funds of knowledge can be extensive. They may include knowledge about building construction, which entails measuring (conversions), masonry, welding, and carpentry; knowledge about gardening, soils, and medicinal herbs; or knowledge about owning and managing small businesses, such as day-care centers, restaurants, and house-painting services.

3. Participants can include two strategies from any of the following categories:

Content-Area Cognates--Cognates are pairs of words in two or more languages that are similar in form and meaning, such as the English word bank and the Spanish word banco. . . . One way for English monolingual content-area teachers to apply this strategy in their classrooms is to invite their English language learners to find content-related cognates as they read in pairs. Students can then compile an ongoing list of cognates found across content-area texts.

Graphic Organizers--Graphic organizers . . . are visual devices that help students quickly access knowledge, tap into prior knowledge, discern relationships, establish a hierarchy among concepts, and review information. When English language learners can simultaneously hear and read content-related information—and see it displayed visually—it helps them decipher the text structures commonly found in textbooks.

Incorporating Multiple Modalities--Because input from multiple modalities benefits most students, especially English language learners, teachers should turn on the English captions when viewing education programs. Combining visual, verbal, and print cues enables English language learners to cross-reference and make meaning using an array of modalities. To help secondary ELL students and striving readers more fully understand a given topic, Heydon (2003) also advocates providing students with a wide variety of texts and genres. These may include picture books, song lyrics, magazines, fiction and nonfiction books, Web sites, graphic novels, newspaper articles, photos, illustrations, and actual objects.

Language Face-to-Face--Teachers should encourage students to engage with one another face-to-face to develop oral and written academic language. . . . several cooperative learning techniques . . . encourage peer interaction. In Student Teams Achievement Division, teams of four or five students are encouraged to interact following a teacher presentation; ideally, the teams are made up of students with varying academic experiences and levels of English language proficiency. In Teams-Games-Tournament, groups of students compete against other groups in various instructional games. In Jigsaw, students research specific topics in small groups, with each group member responsible for one piece of the “jigsaw puzzle.” When groupwork involves students assuming particular roles or taking on specific tasks, it minimizes unequal participation among members of the group, clarifies accountability, and facilitates assessment (Bunch et al., 2001). Group presentations are also a great way to integrate the contributions of English language learners.

Article 3 Title: Celebrating Diverse Minds Questions:

1. What is the central point of the author’s theory that accounts for why students falter in school?

2. Relate the author’s theory to practice by giving an example based on your own experience in working with students fitting this description?

3. What are the limitations of current assessment practices and what needs to be done to address these limitations?

4. In addition to “Do No Harm” Testing Practices, what other recommendations does the author make related to curriculum and for professional development of educators? Recount one recommendation for each of these areas.

5. The author states that … “I am advocating neurodevelopmental pluralism in our schools—the celebration of all kinds of minds. Such an ethos will be the most effective and humane way of realizing our commitment to leave no child behind.” Analyze your own position on this issue and write a statement attesting to your beliefs—whether in full or partial agreement or disagreement with that of the author’s.

Question Responses: Responses should contain information from the following sections or boldfaced sections of text.

1. Participants should state the following:

Many faltering students have specialized minds—brains exquisitely wired to perform certain kinds of tasks masterfully, but decidedly miswired when it comes to meeting other expectations. A student may be brilliant at visualizing, but embarrassingly inept at verbalizing. Her classmate may reveal a remarkable understanding of people, but exhibit no insight about sentence structure. . . . Within every student contending with learning differences, an area invariably exists in which her or his mind has been amply equipped to thrive.

2. Responses will vary.

3. Participants should first explain the limitations and then how to address them:

The methods that schools typically deploy to assess students with learning problems are not up to the task. The discrepancy formulas used to determine eligibility for specialized assistance have been shown repeatedly to have serious flaws (Kavale & Forness, 2000).

Moreover, testing that merely generates a label, such as LD or ADD, accomplishes little. These vague labels do not suggest specific approaches to remediation; instead, they pessimistically imply a relatively permanent pathological condition. What a colossal self-fulfilling prophecy! Most important, diagnosis spawned from a deficit model fails to take into account the most important feature of a student—his strengths.

Assessment for Diverse Minds In addition to rethinking the assessments used to diagnose learning problems, schools need to design regular tests and quizzes so that different kinds of minds can show what they know in different ways. Teachers should

be careful not to tap exclusively rote memory or straight regurgitation of skills and knowledge. They should often allow students to use notes and encourage them to take as much time as they need to respond to questions. . .

High-stakes testing can pulverize many mismatched students. How commonly does end-of-grade testing discriminate against certain kinds of minds? Frequently. As a clinician, I encounter many students who have difficulty performing on multiple-choice tests or operating under timed conditions. These students' dysfunctions in certain skill areas are more than out-weighed by their assets in other domains, but standardized testing never gives them the opportunity to exhibit their strengths. On entering the medical profession, we take an oath that in our practice we will first of all “do no harm.” I offer five suggestions (see “Do No Harm” Testing Practices) to my professional colleagues in education so that they may strive for testing practices that do no harm to students with different kinds of minds. We need to advocate for the elimination of testing practices that inflict needless damage and unfair humiliation on so many students.

4. Responses should address at least one of the following in each area of curriculum and professional development of educators:

Curriculum: If we aspire to meet the challenge of leaving no child behind, we must provide diverse learners with diverging pathways that lead to their success. Such roads should maintain rigorous performance standards, while permitting innovation and creativity in curricular choices and allowing early, highly specialized minds to envision and prepare for productive adulthood.

While advocating ardently for flexibility in achieving the educational aims of schooling, we can still preserve student accountability. No student should be permitted to work, study, or produce less than his or her peers. But we should never insist that everyone put forth identical output.

Professional Development for Educators: . . . continuing education programs abound to help teachers stay abreast of their content, we have found few comprehensive programs devoted to helping educators deepen their expertise in the science of learning. Our not-for-profit institute, All Kinds of Minds, has developed a professional development and school service model called Schools Attuned to help experienced classroom educators become knowledgeable about neurodevelopmental function and variation.1 Participating teachers learn to analyze how their own instructional delivery and content taps specific aspects of memory, attention, motor function, language, and other areas of brain function. They are guided to observe everyday classroom phenomena that open windows on relevant learning processes (Levine, 1994).

Equipped with their Schools Attuned training, teachers lead a coalition involving the student, parents, and other adults in the school to unmask the specific learning profile of a struggling student. With help from professionals trained as neurodevelopmental consultants, whom we call profile advisors (usually school psychologists or special educators), teachers become the primary detectors of

student strengths, weaknesses, and content affinities. The teachers then infuse their insights into their daily group instructional strategies and lesson designs. Frequently, a strategy that they develop to help a particular struggling student benefits the entire class. It's called excellent pedagogy.

Schools Attuned teachers are also committed to making sure that all of their students learn about learning while they are learning. Through a process called demystification, they help students whose neurodevelopmental profiles do not currently mesh with expectations to learn about their own strengths and weaknesses and acquire the terms for the specific processes that they need to work on. With profile advisors as their consultants, regular classroom teachers take the lead in formulating management plans for these students

5. Answers will vary.

Module 8: Technology Applications for School Leaders

Article 1 Title: Data Tools for School Improvement Questions:

1. What is a student information system and what should be examined before its purchase?

2. What are data warehouses and what functions do they perform? 3. What can an instructional management system do? 4. Provide some suggestions given by the author for the purchase of these systems

that you feel would be most critical to maximize their use. Question Responses: Responses should contain information from the following sections or boldfaced sections of text. 1. Student information systems are databases that mainly house demographic data collected throughout the school day. Many student information systems are networked and automate the repetitive collection of such data as class attendance, tardiness, discipline referrals, and enrollments. When buying a student information system, make sure you are clear on how many years of data the system will hold and what the system does with data at the end of each year. 2. Data warehouses enable school districts to analyze data across different databases, such as student information systems, databases of test results and school programs, and databases of information about students' and teachers' perceptions. Schools use data warehouses to perform such statistical procedures as analyzing longitudinal data, disaggregating data, and following cohorts of students over time. 3. Instructional management systems help analyze student performance on ongoing assessments and reveal how closely student learning matches the content a teacher has presented in class. Such systems

• Provide standards-based lesson plans and resources to help students and teachers raise test scores;

• Help teachers align classroom curriculum to content standards; and

• Help teachers align curriculum between grades.

4. Participants can select any of the following:

Be clear about what you are looking for. Be sure the tools can talk with one another. The Schools Interoperability Framework (SIF) is a collaboration of school data stakeholders that sets data exchange standards to enable software packages to communicate with one another. SIF-compliant data tools (which should have this term on the label) interact as one system; data that are entered into one of the data tools will be entered into the other tools automatically.

Involve a team with broad membership in the process. Include teachers, administrators, and information management personnel.

Research possible vendors—and have them come to you.

Don't automatically go to the lowest bidder. Talk with current users.

Article 2 Title: K-8 Virtual Schools: A Glimpse into the Future Questions:

1. What is your understanding of the virtual school for the K-8 learner, as defined by the author?

2. Why would parents choose a virtual school? 3. Do you agree or disagree with the author’s premise that virtual schools address

socialization and diversity? 4. As a school leader, what do you envision in terms of future use and need for

virtual schools for K-8 students?

Question Responses: Responses should contain information from the following sections or boldfaced sections of text. 1. Participants should convey several essential points:

Schools serving younger learners provide a mixture of online and offline activities, with print textbooks and hands-on materials playing a role equal to that of digital technology. In such schools, the computer functions as a management tool that enables personalization of the program for individual learners and real-time collection and use of student achievement data for deep accountability.

Virtual public schools must serve all students who choose to enroll and who are eligible within state guidelines, without regard to ability, previous school performance, or family circumstance. They are available to families at no cost. Like traditional schools, virtual public schools use standards-aligned curriculums, employ certified teachers, and require students to take proctored state standardized tests, often with other students at local school facilities.

As they do in traditional public schools, professional teachers play a central role in K–8 virtual schools, which must meet the same NCLB-defined standards for “highly qualified” teaching staff as any other U.S. public school. That means having the necessary grade-level certifications, as well as subject-area credentials in the upper grades. . . . Homeroom and elementary virtual-school teachers typically have student loads of 40–50 students, but their interactions with students are always one-on-one.

In all K–8 virtual public school programs, a responsible adult, or learning coach, works with the certified teacher to implement the student's learning program. Often this learning coach is the student's parent, but grandparents, aunts and

uncles, caregivers, and others can also perform this role. Although the teacher creates and grades assignments, modifies lesson plans to meet individual student needs, and determines whether the student is ready to advance to the next unit or grade, the learning coach serves as the student's face-to-face motivator and as the eyes and ears of the school.

2. Participants can identify several reasons:

Some students are significantly ahead of or behind their peers; others have learning disabilities or physical health issues. Some students were “bully magnets” at their traditional schools; others were the bullies. Families in isolated rural areas may turn to virtual schools because of transportation issues, whereas families in core urban areas may go virtual out of concern for their children's safety. Young actors, athletes, and musicians may choose a virtual school for the scheduling flexibility it offers. Students who simply need more personal attention in their education program often find virtual schools a good fit.

Virtual schooling customizes the school program to meet each student's particular learning needs.

3. Responses will vary but should center around the author’s claims that on-line teachers encourage extra-curricular activities, online communities link students because of common interests, and that “statistics show that virtual schools attract a cross section of students in terms of ethnicity, home language, socioeconomic status, and special needs.”

4. Responses will vary. Article 3 Title: Foiling Cyberbullies in the New Wild West Questions:

1. How have technologies and their applications spawned what is called a cyberbully?

2. What recommendations are made to school leaders to address cyberbullies? 3. The author states, “Kids need to feel safe everywhere, including in cyberspace.”

Does your school achieve this goal? Analyze the effectiveness of any existing policies in your school with this statement in mind. If your school does not have a policy, draft some ideas for the implementation of one.

Question Responses: Responses should contain information from the following sections or boldfaced sections of text. 1. Respondents should cite that the accessibility of computers connected to the Internet, the availability of mobile phones with text-messaging & camera options, and the ease of access have given rise to the cyberbully--

defined as anyone who repeatedly misuses technology to harass, intimidate, bully, or terrorize another person. Such behavior usually takes the form of inappropriate and hurtful rumors or threats sent through e-mails, instant messages, text

messages, or Web site posts, with the sender often anonymous or disguised as someone else.

2. Responses should contain several of the following recommendations:

Make sure that your school incorporates into its computer instruction clear rules governing the use of technology and warnings for potential offenders.

Make all students aware that no computer or mobile phone—and therefore no person—is really anonymous in cyberspace. Your school's technology teachers should demonstrate to students how each time they access the Internet they generate an electronic fingerprint called an IP, or Internet Protocol address (a string of four numbers punctuated by three periods), which authorities can use to trace all electronic communication from computers or mobile phones . . . behaviors in cyberspace (yes, words are deeds) are downloadable, printable, and sometimes punishable by law. Students need to hear this message, starting in upper elementary school.

Consider enlisting your local police department to help you get the message to students.

Make the rules pertaining to appropriate technology use clear by printing them in student handbooks and in the literature the school sends home to parents, as well as posting them on the school Web site and in every classroom with an Internet-connected computer. A section on cyberbullying should include information about whom to contact if a student feels victimized (see “Sample School Policy Statement on Cyberbullying”). Quickly follow up on all alleged incidents to prevent them from snowballing, and have firm sanctions in place. Kids need to feel safe everywhere, including in cyberspace.

Encourage and highlight professional development opportunities that focus on digital literacy.

3. Responses will vary.

Module 9: School Personnel Article 1 Title: Responding to New Teachers’ Concerns Questions:

1. Compare the population of your school’s teachers with that of the profile of teachers most likely to remain on the job as cited by the researchers. Do you feel any other factors affect teacher retention?

2. After reading the nine major categories of concern expressed by beginning teachers, what concerns from these nine do you consider most critical? Explain how these concerns can be effectively addressed.

3. The authors provide strategies that “can be put in place to smooth the path and make the climate for new teachers more consistent and supportive.” Would the strategies given be easily implemented in your school? Which ones would not be simple to achieve? Explain your responses.

Question Responses: Responses should contain information from the following sections or boldfaced sections of text. 1. Responses will vary but should include data provided by the researchers’ profile of teachers who are most likely to remain on the job:

age 39 to 55; married, with dependent children above age 5; placed in a full-time assignment for which they are highly qualified; and receiving a competitive salary.

2. Participants can list and analyze any of the following concerns of new teachers: • Relationships with students (Will students like me? Will they accept me as a bona

fide teacher?); • Relationships with parents (What will I do if a parent is upset with me?); • Relationships with colleagues (Will my colleagues believe that I know what I am

doing?); • Relationships with supervisors (Will I satisfy the expectations of an evaluator?); • Workload/time management/fatigue (How can I get it all done?); • Knowledge of subject/curriculum (What is really important to teach?); • Evaluation and grading (What am I measuring? What should I do when the

numbers don't match my subjective impressions?); • Autonomy and control (Can I teach in the way that I believe is best?); and • Appearance and identity (How will students judge someone like me?).

3. Participants’ responses will vary but should address the following:

• Provide Reasonable Teaching Assignments • Design Mentoring Programs Carefully • Provide a Comprehensive Induction Process • Build a Network of Collegial Contacts • Design Supportive Evaluation • Prepare New Teachers for Challenges • Encourage Connection to the Profession

Article 2 Title: Bridging the Generation Gap Questions:

1. Compare and contrast the retiring generation of teachers with the new generation of teachers.

2. The authors recommend, “The first step is to understand the makeup of the faculty.” Why is this important?

3. After comparing the strategies provided to integrate the work of new and experienced teachers with your own prior knowledge and experiences, what strategies do you feel would be most effective and why?

4. Conversely, what strategies do you feel might be least effective and why?

Question Responses: Responses should contain information from the following sections or boldfaced sections of text.

1. Respondents can select from any of the following:

The Retiring Generation of Teachers When the cohort of teachers now preparing for retirement entered the profession in the late 1960s and early 1970s, public service was respected and long-term careers were the norm. Throughout society, individuals were expected to pursue just one career—for example, engineering, accounting, or law—and many even dedicated themselves until retirement to a single employer, such as General Electric, Aetna Insurance, or The New York Times. In fact, those who entered teaching at that time were the first cohort to make teaching a lifetime career (Grant & Murray, 1999). Most began teaching right after college, having completed a traditional teacher preparation program. Women and people of color found that the teaching field welcomed them, whereas other professional fields, such as banking and law, presented social barriers to entry. As a result, public schools attracted a talented and committed cohort of new teachers at relatively low expense.

On the job, these teachers have expressed similar preferences. Most of them have chosen to focus their careers on becoming better teachers within the classroom instead of seeking administrative positions beyond it (McLaughlin & Yee, 1988). As a group, they prize the privacy of their classrooms and rely on their colleagues primarily for social support (Johnson, 1990). The egg-crate structure of schools, with each teacher working alone in a classroom, reinforces

these preferences and discourages the development of specialized roles for teachers. The job descriptions for a 30-year veteran and a novice teacher are virtually identical.

The New Generation of Teachers Members of the new generation of teachers enter the workforce in a different context. Fields that once excluded talented women and people of color—such as technology, law, business, medicine, and finance—now actively recruit them. In contrast to public education's low pay and static career path, many of these fields offer high pay, well-resourced workplaces, and opportunities for rapid advancement. The choice to enter teaching today is no longer automatic or obvious. Earning a traditional teaching license is increasingly becoming optional, and today's recruits follow multiple routes to the classroom, such as alternative certification programs or appointments by charter schools. Some entrants have prepared through lengthy university-based teacher training programs with yearlong internships in professional development schools; others have had only six weeks of preservice training in a fast-track program.

As society's career patterns change, young people now routinely anticipate having several careers over the course of their working lives. As one new teacher we interviewed said, “I'm a work in progress. I can't tell you exactly what I'm going to do.” Of 50 new Massachusetts teachers studied by the Project on the Next Generation of Teachers, only 3 were entering teaching as a first career and planned to remain in the classroom full-time until retirement (Peske, Liu, Johnson, Kauffman, & Kardos, 2001).

Moreover, many new teachers today are career switchers. Our random-sample surveys of teachers in six states show that between 33 percent and 48 percent of those entering teaching today come from another line of work rather than straight from college (Johnson & The Project on the Next Generation of Teachers, 2004). Therefore, the conventional image of the new teacher as a young, fresh college graduate fails to fit a significant portion of those entering classrooms today.

Teachers who enter at midcareer often bring with them expectations about their workplace formed on the basis of their experience in other settings. Both they and their younger counterparts tend to be surprised by the isolation of a classroom, expecting instead to learn from colleagues and work in teams. They also hope to have varied responsibilities and gain increasing influence over time but quickly realize that the egg-crate school—with its separate classrooms and uniform teaching roles—does not encourage this kind of growth.

2. Responses can vary but should focus on differing needs due to different career stages, but also include the need to understand the make-up of the faculty in order to

bridge the generation gap and build integrated professional cultures in which new and experienced teachers collaborate regularly and share responsibility for the success of their students and colleagues.

3. & 4. Participants’ rationales will vary but will contain the following strategies:

• Treat the hiring process as the first step of induction. • Assign new teachers to work alongside experienced teachers. • Schedule time for new and veteran teachers to meet. • Provide more than one-to-one mentoring. • Develop school-based induction programs led by experienced teachers. • Organize ongoing professional development on the curriculum. • Encourage teacher leadership and differentiated roles.

Article 3 Title: Sustaining the Pipeline of School Administrators Questions:

1. What is a strength and a danger of tapping leadership talent? 2. If you could design an aspiring leaders program for your school district or your

region, how might it be structured? Ideas given in the article can be used, as well as your own ideas.

3. The authors provide suggestions for internships, as well as for the roles of assistant principal and principal. Select one that you feel is most in need of change and reflect on the proposals made by the authors. What is your evaluation and recommendation for changing that role?

Question Responses: Responses should contain information from the following sections or boldfaced sections of text

1. Responses can contain any of the following strengths and should cite the danger listed below:

Strengths: Tapping leadership talent is a more potent recruitment strategy than the familiar “shotgun” approach. Rather than recruiting leadership talent through broadly targeted mailings, visits, or calls to educators, acting school principals should purposefully identify those teachers or other educators who have clearly demonstrated leadership talent and encourage them to participate in selected leadership activities or administrator orientation programs. The Southern Regional Education Board (SREB) currently uses this strategy in its leadership preparation and development program. The Board has found that schools and school districts are more likely to grant aspiring leaders access to school data and to activities required by their professional preparation program when the schools have previously identified these staff members—or “tapped” them—as having strong leadership potential. Danger: Incumbent administrators may identify as future leaders only those educators whose profiles, values, and behaviors resemble their own. Personnel selection research has demonstrated that we tend to search for candidates who are like ourselves (Pounder & Young, 1996).

2. Participants should include any of the following, as well as other ideas for an aspiring leaders’ program as listed below.

• Familiarize aspiring leaders with various administrative responsibilities and provide leadership experiences with corresponding opportunities for structured reflection.

• Encourage those teachers who demonstrate leadership potential to chair standing committees or special task forces or to coordinate grade-, department-, or school-level initiatives.

• Directly involve aspiring leaders in whole-school planning to give them a broader picture of the many considerations involved in schoolwide efforts.

• Assign aspiring leaders to work closely with diverse student populations, to monitor and analyze these students' learning outcomes, and to report their findings to relevant school and parent groups.

• Provide opportunities for teachers to understand the larger community context by working with school-parent-community groups and related outreach efforts.

• Hone teachers' instructional leadership skills by assigning peer supervision responsibilities.

• Involve teacher leaders in the collection, analysis, and reporting of typical school data that address such accountability issues as school effectiveness or equity of student access and outcomes.

• Include teachers on school leadership teams. 3. Participants’ recommendations will vary based on their selection of roles and their evaluation of that role.

Module 10: Supervision Article 1 Title: Teachers as Walk-Through Partners Questions:

1. How did the principal in this article alter the “Principal's Walk-Through for Reflective Inquiry” model to involve teachers?

2. Explain how the principal got teachers to “buy into” the process and how the

training process was conducted by this principal.

3. What was the goal of this teacher evaluation program when initiated and the benefits that emerged?

4. This principal started with specific departments and trained teachers in various

cycles of implementation. How could this program be initiated in your school to achieve the same goal resulting in similar benefits?

Question Responses: Responses should contain information from the following sections or boldfaced sections of text. 1. Responses should explain that teachers agreed to walk with the principal through their colleagues' classrooms and then meet with the teachers they visited to collegially discuss what they had observed using the model the “Principal's Walk-Through for Reflective Inquiry.” This involved learning

to step into a classroom and quickly identify the teacher's instructional objective, the instructional decisions being made, and the level of cognition being asked of the students (Downey, Steffy, English, Frase, & Poston, 2004).

2. Participants summaries should include the following:

If they agreed to walk with me through one another's classrooms and then come to an after-school meeting with members of their departments to talk about what they had seen, then I would use the drop-in observations I had already conducted to fill out their evaluation forms. They would not have to go through the formal observation process.

I scheduled each teacher for a walk-through session, and the process began. Alone or in pairs, the math and science teachers came into my office during their preparation periods. I quickly introduced them to the walk-through observation process I had learned, and we spent the remainder of the period conducting walk-throughs of 8–10 classrooms together. We stayed in each classroom for only a few minutes—long enough to identify the objective of the lesson, the instructional methodology used, the level of mental engagement required of the students, and the evidence of instructional support on the classroom walls. When we left each classroom, we chatted about what the teachers had observed in relation to these

four points, how it compared with their own teaching, and what insights they had gleaned from their observations.

The following fall, I went back to the veteran, tenured math and science teachers and offered them the same deal with a twist: The math and science departments would walk through each other's classes—math teachers viewing science teachers and vice versa—and then meet collectively. Every teacher agreed.

3. Responses should cite the goal of helping teachers become reflective and see examples of practices they could emulate or question. The benefits include:

We began to notice that teachers talked to one another more about instruction and that teachers had an easier time seeing the logic for pursuing reform. Teachers came to the school administrators to show us how they had added or modified lessons to make them more engaging or challenging for kids. We found that our process gave teachers the opportunities they had craved to visit other classrooms and see what other teachers were doing, as well as guidance in knowing what to look for. Students enjoyed seeing their teachers come into their classrooms, and teachers enjoyed it as well. . . . For the first time, teachers from different departments could understand what instruction looked like in departments other than their own. Perhaps most important, teachers and administrators could talk to one another about instruction, knowing they had common understandings.

4. Answers will vary. Article 2 Title: A Thoughtful Approach to Teacher Evaluation Questions:

1. What is PAR and how does this program work?

2. What benefits were found for new and veteran teachers in PAR?

3. Is PAR a realistic model for implementation in your school? Why or why not?

Question Responses: Responses should contain information from the following sections or boldfaced sections of text. 1. Participants should explain that PAR stands for Peer Assistance and Review and that:

coaches who have been identified for their excellence in teaching and mentoring support new teachers as well as veterans experiencing difficulty in their teaching. The coaches are also responsible for the formal personnel evaluations of teachers in the program. Coaches are usually released full-time from their teaching duties for two to three years so they can provide this support and conduct evaluations. Typically, coaches are not based at a school site, but are matched to teachers across a district on the basis of grade level or subject area.

Some participating teachers receiving the support are new to the profession or the district; others are more experienced teachers who are perceived as needing some form of intervention related to instruction, such as veteran teachers who have received an unsatisfactory evaluation from the principal. Participating teachers typically spend one year in PAR, after which they either successfully exit the program with the principal resuming responsibility for the personnel evaluations or are released from teaching duties in the district.

2. Responses should convey that

participating teachers in the peer assistance and review program reported that they had regular contact with their coaches, with many describing their coaches as accessible, available, and willing to help me when I need it. Moreover, research demonstrates that school districts that implemented peer assistance and review retained more new teachers than they had prior to PAR implementation (Brown, 1993; Hewitt, 2000). Peer assistance and review programs are one promising approach to providing veteran teachers with the help they may need. Principals generally give few unsatisfactory personnel evaluations of teachers (Loup, Garland, Ellett, & Rugutt, 1996; Tucker, 1997). Once a PAR program is in place, however, the number usually increases because, for the first time, a mechanism is available to provide support to veteran teachers who are experiencing instructional challenges.

3. Answers will vary. Article 3 Title: How Walkthroughs Open Doors Questions:

1. What are the overall benefits of walkthroughs? Why do so many administrators fail to utilize them?

2. How are walkthroughs conducted? 3. What questions do you feel are most essential to consider during and after the

walkthrough? 4. Explain how the follow-up dialogue can be kept focused and meaningful.

Question Responses: Responses should contain information from the following sections or boldfaced sections of text. 1. Participants should explain the following: Overall Benefits:

These frequent, short, unscheduled visits can foster focused, reflective, and collaborative adult learning; generally, teachers welcome the opportunities for feedback and discussion that walkthroughs provide. Administrators become more familiar with the school's curriculum and teachers' instructional practices.

• Administrators can gauge the climate of a school: Are students engaged? Are cross-curricular concepts a part of everyday teaching? Are new teachers catching on?

• A team atmosphere develops as teachers and administrators examine instruction and student motivation and achievement together.

• Administrators establish themselves as campus leaders and instructional mentors, influencing teaching, learning, and ongoing school renewal.

• Students see that both administrators and teachers value instruction and learning.

Reasons why they are not utilized: Typically, they neglect walkthroughs because their professional preparation did not include practice in this approach. Administrators still tend to focus on visiting classrooms two or three times a year, following a protocol prescribed by state or district policy. Administrators also face urgent matters requiring their immediate attention every day, so putting off regular visits to classrooms seems justified, and they hope that visiting classes for evaluation purposes fulfills at least a minimum standard for instructional leadership. In addition, most principals do not have coaches to demonstrate the potential benefits of walkthroughs, even in schools where teachers regularly visit one another's classrooms to provide collegial support.

2. Participants should explain the following procedure:

A walkthrough need not be long, certainly averaging no longer than five minutes in each classroom. The key is consistency and commitment. Replacing these 20 minutes with other responsibilities is easy, but the benefits of walkthroughs far outweigh the difficulties of sticking to the schedule.

Walk the entire room. Don't get trapped at the door. A good memory device is to physically touch the back wall of each visited classroom.

3. Responses may contain any of the following questions:

Is there a clear academic focus? Can I ascertain the purpose and expectations of the lesson when I enter the classroom—through what I see on the wall or hear from the teacher and students?

What is the level of student engagement? In general, is the movement, sound, or silence productive? Is student engagement high (80–100 percent), medium (40–79 percent), or low (0–39 percent)? What specific student behaviors indicate the level of engagement?

What do the walls of the classroom show? Is the environment pleasant and innovative? Look for displays of

• Exemplary work,

• Specific scoring criteria (or specific standards and targeted benchmark skills),

• Writing samples with scoring rubrics,

• Helpful information on mechanics of writing (such as capitalization and punctuation skills),

• Helpful information on problem solving, and

• Classroom agreements (ground rules).

How well do students understand the assignment? Select at least two students to speak to quietly and quickly, either at their desks or in the hallway. Ask them some questions:

• What are you working on?

• Why are you doing this work?

• Is what you are working on interesting to you?

• Is what you are working on in other classes interesting to you?

• What do you do in this class if you need extra help?

• Do you have a portfolio? May I see one or two examples of work from this class?

Do students communicate effectively and demonstrate critical thinking skills? Do I see evidence of productive communication styles and higher-order questioning? Can students respond in ways that include personal perspectives and imaginative and thoughtful analyses of new information?

4. Participants should explain that: The administrators and staff should decide in advance on the purpose and format of the walkthrough feedback process. When discussing walkthroughs with teachers, observers may want to mention what they noticed, their questions and compliments, and possibilities for improving equitable and high-quality student learning. In general, informational and supportive feedback is more productive than evaluative comments, with additional coaches or mentors available for more extensive support.

Module 11: School Policy Article 1 Title: Decisions That Have Shaped U.S. Education Questions: Select and summarize the influence of one landmark case on school policy and practice in each of the following areas:

1. Equality in Education 2. Freedom of Speech 3. Student Discipline & School Safety 4. The Role of Religion in U.S. Education

Question Responses # 1 - 4:

Participants should select and explain one of the cases listed in each category as listed and relate its influence on practice and policy in education.

Article 2 Title: Making Inclusive Education Work Questions:

1. Even with amendments and its reauthorization, the premise upon which the Individuals with Disabilities Education Act (IDEA) is based remains the same, state this premise.

2. What are the five systems-level practices recommended for administrators to

successfully make inclusion work?

3. In looking at your own school system, analyze the five practices—which have been successfully implemented and which could be strengthened.

Question Responses: Responses should contain information from the following sections or boldfaced sections of text. 1. Responses should focus on the following:

Since the 1975 implementation of the Individuals with Disabilities Education Act (IDEA), federal law has stated that children with disabilities have the right to an education in the least restrictive environment (LRE). According to the act, removal from general education environments should occur only when a student has failed to achieve satisfactorily despite documented use of supplemental supports, aids, and services.

2. Participants should name the following:

Successful promotion and implementation of inclusive education require the five following systems-level practices: connection with other organizational best practices; visionary leadership and administrative support; redefined roles

and relationships among adults and students; collaboration; and additional adult support when needed.

3. Responses should relate each of the five systems-level practices to their own school system and analyze each in terms of their implementation and level of success. Answers will vary in terms of the analysis. Article 3 Title: No Homework Left Behind Questions:

1. How did the school systematically assess their homework policy, and what was the result?

2. What efforts has Richland Middle School made to promote the after-school lab as a positive experience?

3. How has the goal of making homework a tool for learning been fulfilled through the initiation of the Learning Lab?

Question Responses: Responses should contain information from the following sections or boldfaced sections of text. 1. Participant responses should contain the following steps and end result:

We first asked students to list some of the reasons they might not complete homework. Next, we surveyed our parents (during the beginning-of-the-year open house) to get their perspectives.

Finally, we asked ourselves why we thought homework was important. We brought all staff members—including teachers, aides, and support staff—into this process of examining and enhancing our homework policy. . . . We looked at practices that were working well at Richland regarding homework and how we could expand these successful practices and push through innovations.

Engaging in this process led our group to create the Richland Middle School Learning Lab, an after-school program that would enable us to provide the teacher support and the conditions our students needed to complete their homework.

2. Participants should state that Richland feels it is important that students and the community not view the lab negatively and therefore:

We want students to come for help if they need individual attention to understand a concept, access to computers, and a place to work collaboratively. Teachers make a point of presenting lab sessions as an opportunity to learn, not a punishment. When Richland initiated the program, we mailed all parents an article about the lab, encouraging their questions and support. We also made a presentation to the school board, published an article in the local newspaper, and aired a radio interview.

3. Participants should state that the lab has forced the school to:

examine their homework practices and gain insights and teachers and students have gained insights into how homework can be used to better effect. Teachers started to look closely at the homework we were giving students, asking questions like: What is the purpose of this assignment? Is there another way to show understanding? How much work should we assign? Should we modify assignments for students if they are absent for an extended period of time? This reflection is still a work in progress, but it has brought about positive changes in how staff members view homework practices.

Module 12: School Reform Article 1 Title: Building a MicroSociety

Questions:

1. What are the philosophy and principles upon which the MicroSociety model is based?

2. Explain the structure of the MicroSociety model as implemented at Talbot Elementary.

3. What are some of the challenges and benefits of this program? Question Responses: Responses should contain information from the following sections or boldfaced sections of text. 1. Participants should restate the following:

The MicroSociety philosophy acknowledges that students learn best when they connect schoolwork with the outside world. The following core principles guide the program: promoting student voice and choice, enabling students to learn by doing, taking advantage of students' entrepreneurial spirit, providing real-world experiences, enabling teachers to serve as facilitators, sharing responsibility and authority among teachers and students, and engaging community partners as contributors to the program.

2. Participants should include the following:

Three afternoons each week, every student in our school participates in a for-profit business, a government agency, or a nonprofit organization. Students in grades K–2 work on their MicroSociety activities with their homeroom teachers in their classrooms. Students in grades 3–5 go either to their jobs in various existing organizations or to the small businesses that they have launched as entrepreneurs. If students choose to start a new business rather than join an existing organization, they must create a mission statement, a business plan, and an operating budget and submit them to City Hall (another student-run agency) for approval.

3. Participants should include the following challenges and benefits:

Challenge: The MicroSociety model demands exceptional commitment from teachers . . expanded staff development and time for teacher collaboration are crucial. Benefit: In a model that centers on empowering students, it is essential to encourage leadership at all levels of the organization, from administrators to teachers to paraprofessionals to parents. The bulk of the school's decision making is carried out in ongoing teacher work groups.

Challenge: the lack of adequate resources—has led to amazing opportunities. Benefit: Talbot has applied for and received many grants and awards … in-kind partnerships with local businesses and organizations Benefits: Student academic achievement at Talbot continues to grow. . . .The program positively affects student empowerment, application of real-world skills, and technology skills.

Article 2 Title: Schools That Like a Challenge Questions:

1. What is the balanced school calendar used in some Canadian schools, and how does it maximize student learning time?

2. How does Project CHILD enable teachers to manage instructional time in a different way?

3. Why did the Kennebunk High School initiate the “200 People for 200 Minutes for a Better School,” and what was the outcome?

4. How did the Shorecrest High School address the problem of disengaged students? 5. What programs or ideas could potentially work in addressing an issue needing

reform in your school? Question Responses: Responses should contain information from the following sections or boldfaced sections of text. 1. Participants should include information about the structure of the model and the resulting benefits to student learning.

The model generally includes an alternative school year, a modified school day, and enhanced learning opportunities during breaks. Our students do have school holidays, but these holidays are distributed throughout the year. School starts in early August, five weeks before the traditional start date of early September. The five holiday weeks gained by starting in August are spread out; we have a two-week break in October, a three-week winter break, a week in February and a two-week spring break in March. Staff members and students also enjoy the month of July off.

Beyond reducing the summer learning loss, we strive to use the time that our students spend in school in the most efficient way for learning. We have modified our school day so that two 45-minute nutrition breaks replace the traditional lunch hour and two recesses. Because students spend less time transitioning between classes, they gain instructional time. We estimate that the balanced calendar model adds four to six weeks of instructional time to the school year.

During each break, we offer an inter-session—optional remedial and enrichment classes at an affordable price. Intersessions provide students who need additional help with extra time for learning or a chance to learn through unconventional methods. Classes include a wide range of learning opportunities, from robotics to math to cooperative games.

2. Participants should focus their responses on the restructuring of instructional time as follows:

Students from various designated grade levels are all taught core academic subjects by one teacher designated for that subject, and are sometimes taught in multi-age groupings. Students keep the same teacher for that subject for three years. . . . Rather than the homeroom teacher teaching all subjects, one teacher focused on reading, one on writing, and the third on math, teaching to all grades in the cluster. Students thus had the same teacher for these core subjects throughout 3rd, 4th, and 5th grade.

Each CHILD classroom contained a teacher station for small-group and one-on-one tutorials, a computer station with three to six computers, a textbook station for work with the district's core programs, and three stations for hands-on discovery learning. Each classroom also featured an “exploration station” for manipulating materials, a “challenge station” with games and puzzles, and an “imagination station” for creative projects. A classroom management system, including “passports,” task cards, and daily station assignment boards, kept students focused and on task.

Students rotated among reading, writing, and math classrooms for 60-minute instructional blocks. Science and social studies were taught in the home base classroom. Most blocks began with a whole-group lesson, but follow-up activities took place at the stations. Thus, each student had multiple opportunities for computer-based and hands-on learning in reading, writing, and math.

3. Participants should explain that the school determined that:

Too many students were on the outside looking in, and most had no clearly defined path leading to involvement. We also needed policies to ensure continuation of participatory school governance beyond the good graces of the current administration. . . . In addition, some teachers were struggling with the concept of empowerment, and some parents had only token involvement in school affairs. The school randomly invited approximately 140 students, 90 parents, and all 70 of our teachers a meeting of 200 minutes that were filled with activities that promoted collaboration and inspired an emotional commitment to change.

The result was a tiered school governance structure, as follows:

The first layer offers all stakeholders a voice in school decisions through voting on referendums, responding to surveys, or speaking at schoolwide meetings. The next layer includes the school's main stakeholder groups: our faculty, the parent organization, and our student council (which has open membership). Finally, the school senate, an annually elected body of 8 teachers, 12 students, and 4 parents, makes final recommendations to the school administration.

4. Responses should explain that a long-term strategy of four initiatives was implemented that targeted at-risk students, which included:

Targeted class size reduction. Shorecrest has reduced the student-teacher ratio in all 9th and 10th grade nonhonors English classes from 30:1 to 20:1. We provide teachers in these classes with staff development to help them practice differentiated instruction, targeted writing instruction, and better family communication.

Advancement Via Individual Determination (AVID). This U.S. program nurtures students who are identified as having potential but who need additional academic preparation to go on to college.

Family partnership program. A common feature of many small schools is a strong contract between the school and its families. We are currently assessing the needs of all the families that Shorecrest serves and creating support systems to help parents promote and participate in their children's successes.

Faculty study groups to improve instruction. Inspired by the work of Ginsberg and Wlodkowski,1 we are developing a study-group approach to create highly motivating instruction in all classrooms. Small subject-area teams of three to four teachers will meet weekly to critically examine one another's unit and lesson plans and to analyze student work and student data. They will also observe in one another's classrooms. Pilot projects are scheduled to begin next year, followed by full-scale implementation in 2007–2008.

5. Answers will vary.

Article 3 Title: Pathways to Reform: Start with Values Questions:

1. How does the author justify the importance of values given the emphasis on research-based practices?

2. In the section, “Reflecting on Key Questions” clarify your own beliefs and values by responding to the four questions posed by the author for self- and group-reflection.

3. Even though anxiety may arise when groups discuss values and pluralism, what is the ultimate goal of this exercise?

Question Responses: Responses should contain information from the following sections or boldfaced sections of text. 1. Participant should restate the following:

One crucial but often overlooked source of the distinctiveness among high-performing schools is philosophy—the beliefs and values that create our sense of what makes life worth living, and therefore what is worth teaching and how we should teach it. In our drive to be “research-based,” we tend to forget that between the science of learning and the practice of teaching lie important value judgments that color our reading of the research and the implications for practice we derive from it. These value judgments reflect deeply held philosophical worldviews.

2. Participants should respond to the following: (responses will vary):

• What motivated me to go into teaching?

• What do I think students should know and be able to do?

• Who are the influences on my education philosophy?

• Which colleagues share my vision?

• What do parents, students, and local citizens want, need, and believe?

3. Participants’ responses should reflect understanding that although anxiety may be heightened and that dilemmas may arise with the shift on examining issues through the lens of individual and group values….

The point here is not to create homogeneous communities of value, but rather to create homogeneity with respect to certain core beliefs concerning curriculum, instruction, norms of comportment, and civic virtue. This arrangement leaves plenty of room for students to encounter diverse points of view on substantive matters. Every philosophy of education, every approach to every curriculum, generates disagreements and provides a shared framework for deliberations about them.

Module 13: The School and the Community Article 1 Title: A Community School Questions:

1. Explain what is meant by a full-service school model, as described in this article. 2. Describe the after-school programs that are offered in this school that go beyond

tutoring and homework support, and the resulting goals of these programs. 3. In looking at the array of programs offered in this school, what programs would

be most beneficial in your school to meet the needs of your community? Question Responses: Responses should contain information from the following sections or boldfaced sections of text. 1. Participants should explain that a full-service school . . .

is open year-round to provide a comprehensive program of integrated education, health, and social services.

2. Participants should explain some of the programs such as after-school enrichment classes, before- and after-school child care for school-aged students, and after-school reading and homework clubs that achieve the following goals:

Through affordable programs that are accessible to all students, we provide supervision with a caring adult, support positive social interactions, and improve academic achievement.

3. Answers will vary. Article 2 Title: The Rewards of Parent Participation Questions:

1. Upon what premise is the School Development Program (SDP) framework based? 2. What are some of the ways in which parent participation is made meaningful in

the SDP? 3. What are some of the benefits of the SDP to students, the school, and to the

parents themselves? 4. In studying the SDP framework, can you envision such a model working in your

school? What would have to take place for such strong parent participation to work effectively?

Question Responses: Responses should contain information from the following sections or boldfaced sections of text.

1. Participants should state that

The framework is based on the theory that student academic performance, behavior, and preparation for school and life can be greatly improved when the adult stakeholders work together in a respectful, collaborative way to create a school climate or culture that supports development, good instruction, and academic learning.

2. Responses can contain any of the following:

In School Development Program schools, the existing parent association is rejuvenated as the Parent Team. Through their representatives on the School Planning and Management Team, parents help to create a yearlong, schoolwide schedule of activities designed to support instruction and to create positive relationships in the school. But perhaps more important, they support this comprehensive school plan, particularly its social component. They organize a set of activities to inform parents—discussions about child-rearing, work readiness, how to access out-of-school services, and the like.

The Parent Team also works with teachers to plan the usual back-to-school, Thanksgiving, and other holiday events as purposeful parts of the comprehensive school plan that support students' developmental needs. This differs from other schools, where social activities often have no clear purpose beyond entertainment and therefore are vulnerable to being eliminated in response to pressures for more academics.

3. Participants can cite the following:

The significant body of research suggests that parent participation in school improves the academic achievement of students (Henderson & Mapp, 2002). Additional research shows that life success is more likely when students receive continual, constructive support from meaningful adults who serve as role models and motivators (Adger, 2001).

Highly involved parents can be some of the most effective advocates for education. District administrators and elected officials tend to respond much more positively to school requests when those requests come from parents and staff members together.

Parents with experience in program schools also serve as dependable defenders of teachers.

Another bonus of parent participation in School Development Program schools has been the positive, powerful impact of the experience on the lives of the parents themselves.

Such parents cite as major factors in their success the respect and sense of belonging they received from school staff members. Parents gained confidence as they engaged in school activities. They discovered that they had competencies they were not using, and they also developed new skills. As parents support the activities that the school provides for their children, they are exposed to larger opportunities and they open their minds to their own possibilities. Parental involvement in School Development Program schools not only improves teaching and learning; it can also transform families' lives.

4. Answers will vary. Article 3 Title: It Takes 100 Grandparents Questions:

1. What are some of the programs briefly mentioned in this article that connected seniors to the school?

2. What was the initial goal of implementing the 100 Grandparents program? Explain why this program was important.

3. Can you see this program (or a similar one mentioned) succeeding in your school? Identify at least one program, and discuss the potential benefits implementing a program like this could have for both seniors and students.

Question Responses: Responses should contain information from the following sections or boldfaced sections of text. 1. The following were mentioned and can be included by participants:

The school district launched several efforts to involve senior citizens in schools, including a senior advisory council; "Back to School for Seniors," an annual senior citizens' school visitation event; and "Seniors On Line," computer-assisted instruction for older adults. Another initiative is 100 Grandparents, a program in which senior adult volunteers lead brief read-aloud exercises for K–2 students.

2. This school’s goal was

to improve senior citizens' understanding of public education through participation in meaningful scholastic activities. This involvement, we believe, helps them become ambassadors for students and schools.

It was found that “older voters and voters with no ties to the public school system are among those who turn out to vote in school tax elections in Florida.”

3. Answers will vary

Module 14: Legal Rights and Responsibilities of Students and Staff

Article 1 Title: Hostile Hallways Questions:

1. Explain how the court’s interpretation of sexual harassment is applied in a school setting.

2. Summarize how Title IX, the Fourteenth Amendment, and the Supreme Court ruling in Davis v. Monroe County Board of Education address sexual harassment.

3. After reading the results of the Hostile Hallways survey and the recommendations for schools to address sexual harassment, what could be or is being done in your school setting to deal with this issue?

Question Responses: Responses should contain information from the following sections or boldfaced sections of text. 1. Participants should explain that the courts generally recognize two types of sexual harassment: quid pro quo and hostile environment. These correspond to some aspects of the school environment as well. Quid pro quo harassment occurs when, for example, a teacher offers to raise a student's grade in exchange for a sexual act. By contrast, hostile environment harassment in school includes continual sexual taunting.

2. Participants should summarize key information from each of the following: Title IX of the Education Amendments of 1972 prohibits sexual discrimination, including sexual harassment, in federally funded schools, programs, and activities. States have also enacted statutes against sexual harassment in schools. The Illinois legislature, for example, passed a law requiring schools to create an anti-bullying policy.

The equal protection clause of the Fourteenth Amendment to the U.S. Constitution has been invoked by gay teens seeking relief from sexual harassment. In the case of Nabozny v. Podlesny (1996), . . .

In 1999, the U.S. Supreme Court ruled in Davis v. Monroe County Board of Education that a school district can be held liable in sexual harassment cases when the school knows about the harassment and fails to deal with it adequately.

3. Participants’ responses will vary, but should provide some detail as to how their school can establish, discuss, and enforce a harassment policy, as well as create a supportive environment where …

students feel comfortable talking to adults about harassment and in which students know that staff members will take their complaints seriously.

Article 2 Title: The Right to Search Students Questions:

1. Explain the general interpretation of the Fourth Amendment in school settings. 2. If a student is suspected of carrying an illegal substance, what two conditions

must be met to conduct a search? 3. How may the involvement of law enforcement officials actually complicate the

interpretation of the Fourth Amendment and the standard of reasonable suspicion? 4. Explain the difference between individual versus random searches and the general

legal basis needed for random searches. 5. To avoid legal challenge, debriefing and policy are recommended. How have

these or could these be instituted in your institution?

Question Responses: Responses should contain information from the following sections or boldfaced sections of text. 1. Participants should state that

The Fourth Amendment to the U.S. Constitution guarantees "the right of the people to be secure in their persons, houses, papers, and effects, against unreasonable searches and seizures.” . . . . This right is diminished in the school environment, however, because of the unique need to maintain a safe atmosphere where learning and teaching can occur. Schools must strike a balance between the student's right to privacy and the need to maintain school safety.

2. Participants should apply the Court’s standard of reasonable suspicion in their response by satisfying the two conditions of this standard:

(1) the search is justified at its inception, meaning that there are reasonable grounds for suspecting that the search will reveal evidence that the student has violated or is violating the law or school rules, and (2) the search is reasonably related in scope to the circumstances that justified the search, meaning that the measures used to conduct the search are reasonably related to the objectives of the search and that the search is not excessively intrusive in light of the student's age and sex and the nature of the offense.

3. Participants should explain that while School officials need only reasonable suspicion to search students in public schools, but sworn law enforcement officials normally must have probable cause to search students. Probable cause to search exists when "known facts and circumstances are sufficient to warrant a man of reasonable prudence in the belief that contraband . . . will be found."

4. Responses should convey that

School officials conduct individual searches when they suspect that a student or a small group of students possesses evidence of a violation of the law or school rules. Such searches are subject to the reasonable suspicion standard. Officials conduct random or blanket searches not because of individualized suspicion, but as a preventive measure. Examples of random searches include the use of metal detectors in school entrances and sweeps of parking lots and lockers. The legality of a random search depends on whether the school has a compelling interest or

special need that warrants the use of a search without suspicion. The most common need articulated by schools is the prevention of drug abuse.

5. Responses will vary but should convey some of the recommendations made in the article related to debriefing and policy to guide searches, as follows:

Debriefing. After a search, administrators should meet with those individuals who are involved. Record and reflect on the crucial areas of the search and learn from the reflection. This exercise may be invaluable if the search is subsequently challenged. Document the names of the people who conducted the search; the background of the student who was searched; the alleged infraction; the way the school learned of the infraction; the basis for the search (for example, how reasonable suspicion, probable cause, or consent was obtained); the time and location of the search; the names of the people who were present at the search; and the school policies that were implicated and followed. School officials should also note whether the police were involved or present during the search.

Policy. The best search policies are developed by school boards who work collaboratively with local law enforcement officials, local judges and attorneys, school staff, and community members. A sound policy can make the difference between a legal or illegal search. Sound school search policies should have a mission statement: to maintain a safe learning environment. They should outline techniques for searching students, from the least intrusive to the most intrusive means (metal detectors, canines, breath tests, urine tests, pat downs, strip searches), and they should describe the types of searches students may be subjected to while on school property or at a school function (locker searches, automobile searches, personal belongings, and personal searches). The policies should explain what happens to seized possessions; define consent searches and note how consent may be obtained and the consequences for failing to provide it; state that lockers and other school property are provided for students' use, are under the school's control, and are subject to search at all times; and require that students and parents acknowledge that they have read and understood the school search policy.

Article 3 Title: Copyright 101 Questions:

1. What are some basic facts about copyright law? Share at least two of these. 2. Rewrite the “four tests for fair use” in a user-friendly format that could be shared

with students and staff. 3. In examining the fair use of various materials, which practice do you feel is most

critical to share with students and staff in your school? Question Responses: Responses should contain information from the following sections or boldfaced sections of text.

1. Participants can include any of the following:

First, facts cannot be protected by copyright, but the expression of those facts may be. For example, most of the information in an almanac or encyclopedia is factual and therefore not protected by copyright. The author's choice of words to express those facts, may, indeed, be copyrighted.

Second, notice of copyright is not required. Material is automatically protected by copyright as soon as it is put into tangible form—for example, when it is written on paper, saved to disk, recorded on tape, or painted on canvas. If you can't find a copyright notice, assume that the material is protected by copyright.

Third, copyright lasts for the life of the author plus 70 years for materials created today, but most material published in the United States before 1923 is in the public domain and is no longer protected by copyright.

Fourth, schools have some limited exemptions—known as fair use—to copyright requirements, but the exemptions are for materials used strictly for educational purposes. The exemptions may also be limited in scope, pertaining to only a portion of the material the teacher might wish to use.

2. Responses will vary but should convey the following four tests for fair use:

• The purpose and character of the use. Will the materials be used non- commercially in a nonprofit education institution?

• The nature of the work being copied. Is the work published or unpublished? Is it factual or creative? Unpublished works have stronger protections than do published works. Although facts cannot be protected, the expression of those facts may be.

• The amount of the work being used. Are you using a little, a lot, or all of a work? The more you use, the less likely that the use is fair.

• The effect of your use on the market for or value of the work being copied. What would happen if everyone were to do what you are proposing? Would you deprive the copyright owner of a sale or harm the value in other ways? If you have any commercial intent, even if the money goes to a good cause, harm to the market is assumed (Crews, 2000).

3. Respondents can choose for any of the categories listed and relate it to fair use in their school setting. These categories include: print materials, audiotapes and videotapes, multimedia materials, distance learning, Internet materials, and computer software.

Module 15: School Finance Article 1 Title: A Research Synthesis/Unequal School Funding in the United States Questions:

1. Given that sources of public school funding come from federal, state, and local sources, what is the cause of the differences in funds for wealthy and impoverished school districts?

2. The article states that, “It would surprise most U.S. citizens to learn that disparities [in funding] such as these are simply not tolerated in other developed countries, where public schools normally receive equal funding in rich and poor communities alike on the basis of the number of students enrolled.” Why isn’t this the case in the U.S.? Cite at least two of the proposed reasons given by the authors.

3. In the beginning of the article, the authors pose the following question: “given what we know today, what should we do about inequities in funding for education in the United States?” Given what you learned after reading this article, what do you believe should be done?

Question Responses: Responses should contain information from the following sections or boldfaced sections of text. 1. Response should state that:

Public school funding in the United States comes from federal, state, and local sources, but because nearly half of those funds come from local property taxes, the system generates large funding differences between wealthy and impoverished communities. Such differences exist among states, among school districts within each state, and even among schools within specific districts.

2. Responses should restate any of the following reasons:

Historical Experiences From their beginnings, public schools in the United States have been viewed as institutions that served their local communities.

Beliefs About the Causes of Poverty Resistance to equitable funding for schools has also been supported by several belief systems about the causes of poverty. One of these, the ideology of individualism, holds that success and failure result mainly from individual effort rather than social circumstance. . . . A second belief, essentialism, has it that less-privileged groups (such as African Americans, Hispanics, Native Americans, or women) inherit genetic characteristics that account for whatever lack of successes they have experienced. . . . Another belief system, the culture of poverty thesis, argues that minority persons fail because of inappropriate traditions in the subcultures of their homes, communities, or ethnic groups (see Moynihan, 1969).

When applied to the poor, such beliefs suggest that persons in impoverished communities fail because they possess only “limited linguistic codes” or are handicapped by lack of appropriate “cultural or social capital.”

Flawed Studies Reluctance to provide equal funds for U.S. public schools has also been fueled by claims from prominent researchers, reviewers, and others asserting that the level of funding for schools does not affect student achievement. Not surprisingly, such claims often come from sources that are traditionally hostile to public education.

3. Answers will vary. Article 2 Title: The Matrix Reloaded Questions:

1. What substance is behind the author’s position that No Child Left Behind (NCLB) is an unfunded mandate?

2. In examining the author’s points, are you in agreement or disagreement with his stance on NCLB? Explain.

3. Can equal results occur in the absence of equal funding? After reading this article, formulate a position on this question.

Question Responses: Responses should contain information from the following sections or boldfaced sections of text. 1. Participants may cite any of the following:

No Child Left Behind is an unfunded mandate. The testing provisions alone cost a fortune. After looking into the projected testing costs in seven states this summer, the General Accounting Office, which is Congress's nonpartisan audit arm, issued an alarm about the costs, estimating that the annual national costs of the legislation's testing requirements would run from $1.9–$5.3 billion, depending on whether testing were done on the cheap or the way it should be done. U.S. Department of Education officials, who like to claim that they rely on objective evidence, dismissed the nonpartisan study on the grounds that it involved a sample of just seven states. To a dis-interested outsider, 14 percent of states would seem to be a sample of sufficient size.

Moreover, the study does not include the additional state and local funds required to comply with the many mandates in this legislation, such as developing personalized plans, providing counseling and tutoring, guaranteeing teacher and paraprofessional qualifications, and offering students the opportunity to take public funds to private schools. To meet these mandates, state estimates range from an increase in total funding of 16 percent in New York to more than 80 percent in some southern states.

Funds for Title I have increased 40 percent, federal officials like to say. What they fail to mention is that although the authorization —which is only a promise of funds—increased 40 percent, actual funding has not kept pace. The U.S. Congress and the White House provided only two-thirds of the authorized Title I funding for fiscal year 2003. The White House actually proposed reductions totaling 5.5 percent for overall funding of the Elementary and Secondary Education Act in 2003 and 2004.

2. Responses will vary. 3. Responses will vary. Article 3 Title: To Government Regulations Be True Questions:

1. What are CIPA and NCIPA, and how do these acts affect school funding? 2. What federal funds are tied to these acts? 3. What do these acts require that school districts do to comply in order to continue

to receive federal funding? Question Responses: Responses should contain information from the following sections or boldfaced sections of text. 1. Respondents should define each--Children's Internet Protection Act (CIPA) and the Neighborhood Children's Internet Protection Act (NCIPA)--and explain that these acts affect school districts' eligibility for federal funds. Since almost all schools receive some funds affected by the legislation, they need to comply.

2. Participants should identify the funding sources affected by CIPA and NCIPA. These include:

• E-rate discounts for Internet access and service or internal network connections;

• Funds from Title III of the Elementary and Secondary Education Act that are used to purchase computers to gain access to the Internet or to pay the direct costs associated with accessing the Internet; and

• State-administered monies from the Library Service and Technology Act that fund access to the Internet.

• In addition, school districts are responsible for complying with CIPA and NCIPA if they receive money from an intermediary agency that is administering affected funds, such as a board of cooperative educational services (BOCES), an intermediate unit, or a state department of education.

3. Participants should explain the following:

CIPA requires school districts that receive funding from specified federal sources to use a software-based Internet filtering system to block harmful visual depictions and to adopt and implement an Internet-use policy. NCIPA affects

only those districts receiving funds from the E-rate discount program, a federal program that provides affordable Internet connections to schools and libraries. NCIPA also requires school districts to allow all groups of education stakeholders—such as community groups and parents—to participate in creating the Internet-use policy. School districts that are in compliance with CIPA will be in compliance with NCIPA if they have allowed the public to comment on the district Internet safety policy during the process of creating it.

Module 16: Facilities Management Article 1 Title: A Checklist for Safe Schools Questions:

1. What are some of the physical features of a school building that should be assessed for safety?

2. Explain what types of information should be contained in a crisis management plan.

3. After reading this article, assess your school environment. What areas of strength and what areas of need can you identify in your school facility?

Question Responses: Responses should contain information from the following sections or boldfaced sections of text. 1. Reponses should cite the following as presented in the article’s questions:

How close is the office area to the main access door?

Are the hallways safe?

What communications system is in place?

What traffic patterns are feasible?

What lighting is in place? 2. Participants can use information from the article, as well as information from their schools’ crisis management plan if appropriate, such as:

Your crisis team should have a procedure to notify staff members promptly if a situation occurs during nonschool hours. Along with updating staff home phone numbers, keep an updated list of phone numbers of community support persons and district officials. Remember to conduct crisis-management drills just as you have fire and disaster drills. Select a hypothetical situation and then proceed with your crisis-management plan accordingly.

3. Answers will vary. Article 2 Title: Small Classes, Small Schools: The Time is Now Questions:

1. What has caused issues of school and class size to resurface again? Cite at least three reasons.

2. Compare the proponents’ claims regarding the benefits of consolidated schools in contrast to the latest research findings on these large schools.

3. Compose a persuasive paragraph to convince others of your position on this issue.

Question Responses: Responses should contain information from the following sections or boldfaced sections of text. 1. Participants’ responses can include any of the following:

First, the standards movement has encouraged the resurgence of the class size and school size debates. The standards movement has highlighted the fact that schools are largely inequitable places. Students in schools with large populations of disadvantaged students perform least well on standardized assessments. Evidence also suggests that these schools often have the least-experienced teachers (NCTAF, 1996; Roza, 2001). In effect, having standards in place emphasizes that standards are necessary but insufficient in themselves to improve student performance. Unless we change students' learning opportunities, especially for students who are ill-served by their schools, standards alone are unlikely to influence student learning.

Second, class size and school size issues have resurfaced because of the increasing consensus among educators and the public that all students can learn. When I began teaching in the early 1970s, teachers generally accepted the notion that

Third, following the events of September 11, educators have a renewed appreciation for the importance of the basic freedoms we enjoy and the advantages that a democracy provides its citizens. We know that a democratic citizenry must value differences among its participants. Schools should strive to develop in students the skills that they need to examine their

Renewed interest in class size and school size is broad-based and nationwide. The Bill & Melinda Gates Foundation has dedicated more than $250 million to reducing the size of U.S. high schools. The U.S. Department of Education has committed $125 million to fund small-school initiatives. In Boston, Chicago, and New York, small-school initiatives are under way. Small-school collaboratives, designed to support the change from comprehensive high schools to smaller learning communities, are springing up everywhere and include New Visions for Learning in New York, the Small Schools Workshop in Chicago (Illinois), the Small Schools Project in Seattle (Washington), and the Bay Area Coalition of Essential Schools in Oakland (California).

Lawmakers in Kentucky, California, Georgia, and Washington have passed legislation to reduce class sizes, believing that teachers will be better able to help all students meet the standards when the teacher-student ratio is substantially reduced.

2. Responses should convey that

proponents of the school consolidation movement suggested that schools would be more efficient and effective if they were larger. Single plants housing 500–2,000 students presumably could offer greater variety in subject matter, would provide teachers with the opportunity to track their students according to

ability, and might put less strain on community resources (Wasley & Fine, 2000).

Research conducted on the validity of the assertions favoring large schools has suggested that less-advantaged students end up in the largest classes, with the least-experienced teachers and the least-engaging curriculum and instructional strategies (Oakes, 1987; Wheelock, 1992). Further research suggests that schools are organized more for purposes of maintaining control than for promoting learning (McNeil, 1988).

3. Responses may vary but should be based on facts.

Article 3 Title: Common Space, Common Time, Common Work Questions:

1. How can beliefs about professional development influence the use of space, time, and work?

2. How do common work spaces support new teachers? 3. What other facets of the total school design are essential to support and maximize

the concept of shared space? Question Responses: Responses should contain information from the following sections or boldfaced sections of text. 1. Respondents should explain that in the high school cited in this article,

three organizational structures—common space, common time, and common work—make professional development of both new and experienced teachers a natural part of teachers' work. These structures are also grounded in the school's founding beliefs. The school's belief statement describes teacher learning as a lifelong, active pursuit, involving problem solving and the performance of authentic tasks. Teachers are viewed as professionals who deserve support from the community and the administration for their learning and growth. They are expected to collaborate with colleagues, students, parents, and community members.

2. Participants should explain that

These common workspaces are invaluable in helping new teachers make sense of teaching. As a new social studies teacher remarked, the space enables him to listen in on veteran teachers' spontaneous conversations about teaching dilemmas and to learn from their deliberations and experiences. This is important because sometimes, he said, new teachers don't know what questions to ask. . .

“Leaning over” to ask for help occurs easily in the workspace that most teachers share with their colleagues

3. Responses should contain other facets such as

. . . who will share office spaces. Coteaching teams, cross-curricular grade-level teams, and content-area teachers are clustered together. Consequently, teachers who teach students in the same grade level can easily consult one another as issues arise. Teachers who teach the same courses can sit together as they plan units and lessons. Veteran teachers can help novice teachers.

Although common workspace is an important organizational structure, it would be unproductive if it were not combined with common planning time. Many structural features of the school curriculum create common tasks for teachers that require collaboration. In Poland Regional High School's highly collaborative culture, learning to teach requires new teachers to have the skills and the desire to work closely with their colleagues. The high school specifically looks for these qualities in candidates during its hiring process.

Module 17: The Principalship Article 1 Title: The Best Way to Lead Them Questions:

1. Explain how creating an environment for learning is both external and internal.

2. After reading the section on “Cultivating a Professional Community,” evaluate how this can be achieved or enhanced in your own educational institution.

3. This article contains a quote that, “Leadership begins at one's center: authentic

leaders build their practice outward from their core commitments, . . . ” What are your core commitments?

Question Responses: Responses should contain information from the following sections or boldfaced sections of text. 1. Participants should arrive at the conclusion that the physical beauty of a school—in that it is clean and pleasing to the eye--translates into an internal environment that is conducive for learning.

2. Participants should evaluate how to create an environment in which

teachers collaborate, exchange ideas, and develop tight collegial connections—and in which principals share governance with their staff members

3. Responses will vary.

Article 2 Title: A Framework for Shared Leadership Questions:

1. How can leadership capacity be developed in teachers? Select two methods that you evaluated to be most effective.

2. What features were found in the research to be vital to school improvement?

3. Using information from your reading and your own experiences, explain whether

you agree or disagree with the conclusion reached at the end of the article stating, “Our mistake has been in looking to the principal alone for instructional leadership, when instructional leadership is everyone's work. We need to develop the leadership capacity of the whole school community. Out of that changed culture will arise a new vision of professional practice linking leading and learning.”

Question Responses: Responses should contain information from the following sections or boldfaced sections of text. 1. Participants can cite and explain any of the following:

• Study Groups--Study groups read articles or books together and discuss the implications of the texts' ideas.

• Action Research Teams--Action research teams identify a compelling question of practice and conduct research to discover information that will shed new light on the question and lead to new actions.

• Vertical Learning Communities--In the vertical learning communities model, multiple grades are linked together in a common community in which teacher leaders have the authority to work closely with students in instruction, curriculum design, discipline, and family relations.

• Leadership Teams--Teachers can . . . “analyze data and to plan, advocate, monitor, and implement the school improvement plan.” . . . “strong implementation teams [are] armed with authority, resources, support, and time.

• An Integrated School Improvement Process--Participation is most powerful when combined into a thoughtful and integrated school improvement process.

2. The features include:

skillful participation, vision, inquiry, collaboration, reflection, and student achievement that interact to create the new tasks of shared instructional leadership.

3. Responses will vary.

Article 3 Title: Growing into Leadership Questions:

1. The authors state, “One of the difficulties that new principals face is that they must lead while they are learning to lead.” Based on what you have read and your prior knowledge and experiences, what support structures would you recommend for new principals?

2. Select one or more of the themes that resonate most closely with your own beliefs about leadership, and express why you feel they are vital to maintain.

3. Given the present focus of education, which theme would be hardest to maintain? Explain why.

Question Responses: Responses should contain information from the following sections or boldfaced sections of text. 1. Responses will vary. 2. & 3. Responses will vary but the themes that are discussed include:

Keep Students at the Heart Be a Learning Leader Act Ethically Put Instructional Leadership First Practice Efficient Management Build Strong Relationships Know What to Expect Orchestrate School-Community Partnerships Lifelong Learners

Module 18: The Superintendency Article 1 Title: New Lessons for Districtwide Reform Questions:

1. How can effective district leaders use the key advantages of their position to implement their vision?

2. Explain what the authors mean when they say the collective moral purpose is put into practice by the superintendent.

3. Why is capacity building essential? 4. Based on the 10 components of effective district leadership presented in this

article, articulate your philosophy of leadership at the district level. Question Responses: Responses should contain information from the following sections or boldfaced sections of text. 1. Responses should contain the following:

Aside from an understanding of the education field's new insights into pedagogy and change, effective district leaders also know how to use the key advantages that their position affords them for implementing their vision: They have the mandate from the board that appointed them, the big picture of the organization because of their position in it, a public forum and visibility, and control over the financial and human resources necessary for bringing about change.

To implement their vision, district leaders must build a coalition of leaders who pursue the vision in practice. Like distributed leadership at the school level, large-scale reform requires pluralized leadership, with teams of people creating and driving a clear, coherent strategy. The district will also require external support, but change will not be possible without this daily, internally driven leadership. Having a driving conceptualization means high engagement with others in the district and plenty of two-way communication that deepens shared ownership and commitment.

2. Participants should explain that

Collective moral purpose makes explicit the goal of raising the bar and closing the gap for all individuals and schools. That moral imperative applies to adults as well as to students. We cannot advance the cause of students without attending to the cause of teachers and administrators. Many passionate, morally driven superintendents have failed because they blindly, even courageously, committed themselves to students, running roughshod over any adults who got in the way.

3. Participants should explain that capacity building is

The main mark of successful leaders . . . the number of good leaders they leave behind who can go even further. . . . leaders need to focus on both achievement

and the development of future leaders. Their own professional development must continue throughout their careers, especially in learning how different roles in complex, uncertain environments work in tandem. This learning needs to occur in context—not only through workshops but also through daily interactions in cultures designed for job-embedded learning and coordination. Capacity building is essential for everyone: school-level, area-level, and system-level employees, including the most senior executives.

4. Responses will vary but should be based on the 10 crucial components of effective district leadership and reform, which include:

a compelling vision, collective moral purpose, effective structure, capacity building, lateral capacity building, ongoing learning, productive conflict, a demanding culture, external partners, and growing financial investment.

Article 2 Title: A Public Agenda Survey / Staying Ahead of the Game Questions:

1. What are some of the major issues cited by superintendents that interfere with their ability to do their job?

2. In reviewing the ways to improve school leadership, do you agree or disagree with some or all of the claims cited?

3. In reviewing the implications for schools and districts, what do you evaluate as the most critical area needing to be addressed to retain the “can do” spirit of practicing school leaders?

Question Responses: Responses should contain information from the following sections or boldfaced sections of text. 1. Respondents may cite any of the following:

Eight in 10 superintendents (81 percent) say that politics and bureaucracy are the main reasons that colleagues leave the field. Problems such as low pay (5 percent) or the pressures of higher standards (10 percent) do not even come close. More than half of school superintendents (54 percent) say that they need to “work around the system” to get things done. Ten percent say that the system actually ties their hands. Sixty-nine percent of superintendents say that the school board sometimes interferes with their ability to do their job. Half of superintendents say that legal issues (50 percent) and complaints from parents and special interests (48 percent) take up too much of their time. Almost as many (43 percent) say that issues related to the unions and collective bargaining are time-consuming.

2. Answers will vary.

3. Answers will vary.

Article 3 Title: The Invisible Role of the Central Office

Questions:

1. Explain several roles that the central office fulfills in the district as described by the author.

2. How does the role of the central office described in this article compare with that of your district’s central office?

Question Responses: Responses should contain information from the following sections or boldfaced sections of text. 1. Responses should convey that the central office enables the district to focus on priorities and that the central office works to communicate those messages to schools and community. The central office fosters teacher leadership through programs, like lead teachers, and conference presentations and attendance. They orient new teachers and provide service and expertise. They also ensure consistency as follows:

How does a school system set goals and work toward them? How does a school system avoid a balkanization of its neighborhoods and schools? How can families trust that their children will receive a high-quality instruction program at any of the schools in a school district?

Through the central textbook adoption process, the development of curriculums, the publication of programs of studies, the development of services to special populations, classroom observations and feedback, staff development activities, and community forums, we establish ways to achieve the district's goals and consistency of the instruction program. Certainly, school staff members vary the instruction program to better meet the needs of their students and to take advantage of their own strengths and interests.

2. Answers will vary.